SlideShare ist ein Scribd-Unternehmen logo
1 von 299
PRIMER SIMULADOR CURSO
  ENARM CMN SIGLO XXI
  INICIO DE LA SEGUNDA PARTE
CASO CLINICO
Se trata de paciente de 49 años con perdida de peso de
  mas de 15 % en 4 meses, acompañado de ictericia y
  dolor en la región epigástrica, tiene el antecedente
  de alcoholismo de 30 años de duración, tabaquismo
  positivo, hace 2 años fue tratado en varias ocaciones
  por gastritis, a la EF se palpa vesícula no
  dolorosa, indurada, se observa con tromboflebitis.
PREGUNTA No. 251
Considerando el diagnostico presuntivo que ha
  considerado, cual es signo mas frecuente observar?

RESPUESTA
a.- Ictericia.
b.- Perdida de peso.
c.- Vesícula no dolorosa.
d.- Esteatorrea
PREGUNTA No. 252
Considerando el diagnostico presuntivo que ha
  considerado, cual es síntoma mas frecuente
  observar?

RESPUESTA
a.- Distención abdominal.
b.- Dolor abdominal epigástrico sordo.
c.- Dolor abdominal epigastrico difuso.
d.- Prurito generalizado.
PREGUNTA No. 253
Considerando el diagnostico presuntivo cual es el
  estudio de elección para apoyar el diagnostico?

RESPUESTA
a.- IRMf.
b.- TAC helicoidal.
c.- USG.
d.- Gamagrama.
PREGUNTA No. 254
Considerando el diagnostico considerado por usted,
  cuales es la metástasis primaria mas frecuente?

RESPUESTA
a.- Hepático.
b.- Ganglios linfaticos.
c.- Peritoneo.
d.- Pulmón
PREGUNTA No. 255
Considerando las caracteristicas e incidencia cual es la
  neoplasia mas probable?

RESPUESTA
a.- Cistoadenoma mucinoso
b.- Adenocarcinoma pancreatico
c.- Cistoadenomas ceroso.
d.- Neoplasia quistica solido papilar.
CASO CLINICO

Se trata de paciente femenino de 65 años de edad que
  acude a consulta por dolor difuso, perdida de
  peso, refiere que ha presentado cuadros de
  mareo,     piel    fría,   sudoración,     frio,    con
  desorientación, durante estos cuadros ha disminuido
  los síntomas con el consumo de dulces, a la EF se
  destaca taquicardia, diaforesis y glucosa periférica de
  50.
PREGUNTA No. 256
El paciente presenta la triada de wipple, cual es el
  método diagnostico inicial para considerar su
  localización?

RESPUESTA
a.- USG.
b.- TAC.
c.- IRM.
d.- Doppler transoperatorio.
PREGUNTA No. 257
Cual de los siguientes padecimientos no es diagnostico
  diferencial?

RESPUESTA
a.- VIPoma.
b.- Gastrinoma
c.- Insulinoma.
d.- Papiloma
CASO CLINICO
Se trata de paciente masculino, que acude por
  cansancio, fatiga, así como dolor en hipocondrio
  derecho, anemia aguda, se realizo USG de hígado
  y vías biliares donde se apreciaron cálculos
  biliares, no se aprecia ictericia sin embargo, se
  realizo estudios de laboratorio observando
  glucosa de 190, el paciente refiere que no era
  diabético,             Anemia        normocromica
  megaloblastica, durante su estudio el paciente
  perdió mas de 20 % de su peso y su estado se
  deterioro severamente, su piel se observa
  aspera, esteatorrea e hipocloridria.
PREGUNTA No. 258
Cual es la conducta mas apropiada a seguir?

RESPUESTA
a.- Dieta hiperproteica y complejo B.
b.- Hierro suplementario y complejo B
c.- Suplementacion enzimatica.
d.- Tomografia axial computada.
PREGUNTA No. 259
Considerando los datos clínicos, usted solicito valores
  de somatostatina encontrándose en 130 mcg/dl, se
  diagnostico somatostinoma, cual es la conducta?

RESPUESTA
a.- Quimioterapia y posterior resección.
b.- Embolización y radioterapia.
c.- Realizar Wipple.
d.- Braquiterapia previa em
CASO CLINICO

Se trata de paciente de 7 años con dilución, disfunción
  y atrofia, edema caída de cabello, anorexia y
  diarrea,                                     lesiones
  queratosis, seca, ictiositosica, además lesiones
  hipercromica, lesiones purpuricas, asi como
  petequias, piel marmorea con telangiectacias.
PREGUNTA No. 260
Cual es su impresión diagnostica inicial?

RESPUESTA
a.- Purpura.
b.- Leucemia.
c.- Desnutrición.
d.- Fibrosis quistica
PREGUNTA No. 261
Considerando el protocolo de estudio para apoyar su
  diagnostico cuales son los estudios de laboratorio y
  gabinete mas adecuado?

RESPUESTA
a.- BH, EGO, ES, PFH, COPRO.
b.- EGO, ES, BH, PFR, PFH.
c.- BH, Frotis de sangre, EGO, Cultivos, PFH.
d.- Perfil lipidos, Frotis de sangre, EGO, BH, PFH.
PREGUNTA No. 262
Cual es la manifestación inicial de laboratorio que se
  presenta en el paciente con desnutrición?

RESPUESTA
a.- Anemia.
b.- Hipoalbuminemia.
c.- Leucopenia.
d.- Plaquetopenia.
PREGUNTA No. 263
El paciente presenta en una segunda consulta las
  siguientes manifestaciones, diarrea, anorexia
  palidez, sin ganas de jugar, que tipo de
  manifestaciones son?

RESPUESTA
a.- Dilucionales
b.- Circuntanciales
c.- Agregados.
d.- Comorbidos
CASO CLINICO
Paciente de 15 años Presentaba hipogonadismo
  hipogonadotrófico, retardo mental y IMC=57,7
  kg/m2. El paciente presentaba serios problemas de
  higiene, micosis en pliegues y múltiples problemas
  sociales. Había intentado a lo largo de su vida
  variados tratamientos médicos para reducción de
  peso (dietas, programa de ejercicio físico, uso de
  anfetaminas, etc.) sin éxito permanente, ya que
  debido al componente hiperfágico de su síndrome de
  base no lograba mantener la disminución ponderal.
PREGUNTA No 264
Considerando las manifestaciones clínicas, se realizo
  un cariotipo 46 XY del (15) (q11q13)?

RESPUESTA
a.- Prader – willi.
b.- Sindrome de carpinter.
c.- Sindrome de cohen.
d.- Sindrome Down.
PREGUNTA No 265
Considerando el cuadro clínico, cual es la conducta a
  seguir?

RESPUESTA
a.- Dieta
b.- Ejercicio
c.- Estilo de vida.
d.- Y de Roux
PREGUNTA No. 266
Cual es el tratamiento farmacológico a largo plazo con
  evidencia y aprobación para el paciente
  considerando su edad?

RESPUESTA
a.- Fentermina
b.- Metfomina
c.- Sibutramina
d.- Orlistat
CASO CLINICO

Una mujer de 45 años presenta dolor en hipocondrio
  derecho de 36 hrs. de evolución. TGP:
  80, amilasemia: 250 ui. La ecografía abdominal
  informa: quiste de 3 cm de diámetro en segmento III
  del hígado, pared vesicular de 5 mm trilaminar, con
  un diámetro vesicular transverso de 106 mm y un
  cálculo impactado en cuello.
PREGUNTA No. 267
Cual es la conducta a seguir mas adecuada?

RESPUESTA
a.- Hidratación, analgésicos, antibiótico y derivación por
    consulta externa para cirugía programada
b.- Hidratación, analgésicos, antibiótico y cirugía abierta
    de urgencia.
c.- Hidratación, analgésicos, antibiótico y laparoscopía a
    partir de las 72 hs de internamiento.
d.- Hidratación, analgésicos, antibiótico y laparoscopía
    antes de las 72 hs de internamiento.
CASO CLINICO
Una paciente de 74 años, apendicectomizada a los 10
  años y sin antecedentes médicos de importancia
  ingresa a la guardia por vómitos, distensión
  abdominal y falta de eliminación de gases de 48
  horas de evolución. En la radiografía de abdomen
  (ver imagen), el cuadro del paciente se ha presentado
  recurrentemente en los 5 últimos años.
RESPUESTA No. 268
¿Cuál es la patología obstructiva más probable en
  este cuadro?

RESPUESTA
a.- Cáncer de colon derecho
b.- Bridas de intestino delgado
c.- Fecaloma
d.- Constipación crónica pertinaz
CASO CLINICO
Se trata de paciente masculino de femenino de 36
  años de edad, con sobrepeso que reingreso
  posterior a un mes de tratamiento por quemaduras
  de 3 grado, acude con dolor en hipocondrio
  derecho,     el     cual   se     activa    con     la
  alimentación, predominantemente con grasas, el
  ultrasonido se muestra a continuación. A la
  exploración física se encuentra estable signos vitales
  dentro de rango de normalidad, solo destaca
  murphy positivo, con puntos uretrales altos
  igualmente positivos, PFH sin datos patológicos.
PREGUNTA No. 269
Cual es la conducta a seguir?

RESPUESTA
a.- Realizar cirugía laparoscópica.
b.- Hidratación, analgesia, antibiótico.
c.- Realizar CPRE.
d.- Alta y envío a consulta externa de cirugía.
PREGUNTA No. 270
Cuales son los agentes infecciosos más frecuentes que
  se pueden encontrar en el presente caso?

RESPUESTA
a.- Eschericia coli.
b.- Klebsiella penumoniae.
c.- Steptococcus faecalis.
d.- Bacteroides fragilis
PREGUNTA No. 271
El paciente inicia con fiebre súbita de 39.7 grados, con
   ataque al estado generalizado, dolor en hipocondrio
   derecho que se irradia a en barra y es de características
   trasfictivo, se observo un incremento de DHL, PCR y VSG
   así como leucocitosis de 21,000 e hiperglucemia de 250
   mg/dl, cual es la conducta mas adecuada a seguir?

RESPUESTA
a.- Realizar LAPE.
b.- Hidratación, morfina y antibiótico.
c.- Realizar CPRE.
d.- Realizar TAC.
CLINIC CASE

An 80 year old man has a 20 month history of
  progressive gait disturbance and incontinence, which
  had been attributed to old age and prostatism. His
  gait is dhort-stepped, and he turns very
  slowy, almost toppling over.
QUESTION No. 272

The most likely diagnosis is?

ANSWER
a.- Chiari malformation.
b.- Cruetzfeldt-Jacob disease.
c.- Progressive multifocal leukoencephalopathy.
d.- Normal pressure hydrocephalus.
CLINIC CASE

A 29 year old man presented with sudden onset low
  back pain with radation to his right leg while
  attempting to lift a box. The neurological exam
  showed normal muscle strength in his legs bilaterally
  with preserved anal tone.
QUESTION No. 273
The best management at this point is?

ANSWER
a.- Acumpunture referral.
b.- Bed rest with NSAID´s.
c.- CT scan of the lumbar spine.
d.- Stat MRI of the lumbar spine.
CLINIC CASE

A 52 year old female presented to the emergency room
  with sudden onset of mental status change
  associated with left sided weakness. Initial CT head
  without contrast showed right temporal intracerebral
  hemorrhage.

  CT angiogram of the head is shown below.
QUESTION No. 274
Based on the imaging findings, what is the likely
  etiology for this intracerebral hemorrhage?

ANSWER
a.- Hemorrhagic conversion of ischemic stroke.
b.- Posterior comunicanting artery aneurysm.
c.- Hypertensive cerebral hemorrhage.
d.- Dural sinus thrombosis.
CLINICAL CASE

A 70 year old male was brougth to the emergency room
  by his wife to be evaluated for his agitation. The
  patient went to bed last night with normal behavior.
  He awoke this morning confussed with visual
  hallucinations.

The patient was found to have an ischemic stroke
  associated with agitated delirium.
QUESTION No. 275
Which of the following arteries is responsible for his
 stroke?

ANSWER
a.- Left anterior inferior cerebral artery.
b.- Left middle cerebral artery.
c.- Left posterior cerebral artery.
d.- Left posterior inferior cerebellar artery.
CLINIC CASE

A 70 year old patient with past medical history of
  hypertension and diabetes, complains of sudden
  onset visual disturbance.

Neurological examination reveals right homonymous
  hemianopia with macula sparing.
QUESTION No. 276
What is the localization of this ischemic stroke?

ANSWER
a.- Left occipital lobe.
b.- Right occipital lobe.
c.- Left parietal lobe.
d.- Right parietal lobe.
CLINIC CASE

A 55 year old man had a transient episode of left
  sided weakness that lasted 45 minutes. The patient
  was evaluated at the emergency department with
  a CT scan of the head w/o contrast, which was
  normal.
QUESTION No. 277
What factors should be considered regarding the
 decision for impatient vs outpatient workup?

ANSWER
a.- AII TIA patients should be worked up as outpatient.
b.- ABCD2 score should be considered.
c.- The patient vs outpatient workup is the
   neurologist´s preference.
d.- No need for extensive workup as the symptoms
   resolved completely.
CLINIC CASE
An 83 year old woman with hypertension and atrial
  fibrillation on coumadin presented with an acute
  onset of left-sided weakness. Her daughter called an
  ambulance.

The patient was evaluated in the emergency room and
  CT head was obtained (see image).
QUESTION No. 278
What is the diagnosis?

ANSWER
a.- Subdurachnoid hemorrhage.
b.- Intracranial hemorrhage.
c.- Subdural hematoma.
d.- Epidural hematoma.
CLINIC CASE
A 4 year old male presents to your office for evaluations of
  his high fever. He was seen 2 days ago at your office with
  ear pain and a temperature of 100.5 F. At that time, noth
  tympanic membranes were bulging, erythematous, and
  opaque. He was prescribed ab oral antibiotic for bilateral
  otitis media.
The mother states that she was unable to get the
  prescription filled until this morning, Vital signs in the
  ofice include an oral temperature of 39.4 C. Both heart
  rate and respiratory rate are increased, but not over
  what would be expected based on the degree of fever.
  Otoscopy finding are unchanged. The left pinna is noted
  to be protruding laterally.
QUESTION No. 279
This patient is most likely to experience tenderness to
  palpation in which of the following anatomic areas?

ANSWER
a.- Left mastoid region.
b.- Left paranasal sinuses.
c.- Left temporal region.
d.- Left lower molars
CLINIC CASE

A 3 year old females is brought to your office for
  evaluation of fever and a rash. She was in her usual
  state of goog health until the day before
  yesterday, when she developed a fever to 38.6 C.

For the past two days she has rested more than usual
  and been fussy but consolable.
Last night the mother noticed red bumps on her face.
  This morning those lesions looked like fluid-filled
  blisters, and she has new red bumps on her chest
  and upper arms.

The mother tells you that she has tried to kepp her
  daughter from scratching them because she doesn´t
  want them to spread. She has scratched one of the
  lisions on her face so much that if has broken open
  and crusted over.
QUESTION No. 280
Which pf the following represents the most
 appropriate management of the patient at this
 time?

 ANSWER
a.- Caution the mother not to become pregnant
   during the childs illnes.
b.- Assure the parent that the rash will resolve on its
   own without specific tratment.
c.- Prescribe asperin to control the fever.
d.- Prescribe oral ketoconazole and penicillin.
CLINIC CASE

A 3 year old female presents with a fever of 102.7
  F, chills, and vomiting. She had been in her usual
  state of good health until yesterday, when she
  became ill with what her parents thought was a
  gastrointestinal illnes.
Review of systems is negative for rhinorrhea, cough, ear
  pain, sore throat, diarrhea, and rash. Her past
  medical history is significant for 2 prior episodes of
  cystitis, both of which resolved with oral antibiotics.
Micrioscopic examination of the urine reveals
 numerous white blood cells, red blood cells and
 gram negative rods.

Elevation of the erythrocyte sedimentation rate
   suggests involvement of the upper urinary tract.
   The patient is admitedd to the hospital for
   intravenous and fluids.
QUESTION No. 281
Which of the following is the most likely cause of this
 patient´s recurrent urinary tract infections?

ANSWER
a.- Polycystic kidney disease.
b.- Posterior urethral valves.
c.- Vesicoureteral reflux.
d.- Primary b-cell inmunodeficiency.
CLINIC CASE

An 8 year old boy was at an end of the year school
  picnic when he was stung by a flying insect. Within
  minutes of being stung, the youngster began to have
  difficulty breathing. Paramedics were called to the
  scene.
The frightened child was found to be moderately
  hypotensive, wheezing, and in significant respiratory
  distress. Epinephrine was administered at the
  scene, and the patient was placed on axygen via face
  mask.
Upon arrival to the emergency department, the patient
  received a dose each of an intravenous antihistamine
  and intravenous corticosteroid.
He was observed overnigth in the hospital and
  discharged the next day after the parents had
  obtained injectable epinephrine and been taught to
  administer it.
He is in your office today for futher education and
  follow-up care. He has a positive skin prick test for
  yellow jacket venom.
QUESTION No. 282
Which of the following represents the most appropriate
 next step in the management of the patient?

ANSWER
a.- Daily administration of a non-sedating antihistamine
   throughout the spring, summer an fall months.
b.- Skin prick testing for hornet venom sensitivity.
c.- Allergen immunotherapy.
d.- Specific serum honeybee IgE antibody testing.
CLINIC CASE

A 5 year old female is in your office for her yearly
  health maintenance visit, She is in her usual state of
  good health. Height and weight continue to track
  along pre-estabilished curves, both at the 50th
  percentiles, her mother describes her as active and
  curious.

While examining her chest, you note a mid-systolic
 grade II/VI vibratory murmur heard best at the left
 lower sternal border.
The murmur intensifies in the supine position. There
  are no associated heaves, thrills, clicks or rubs. The
  second heart is normally split with inspiration and
  becomes singles on full expiration.

Femoral pulses are symmetric and not delayed. No
  cyanosis or clubbing is noted on examination of the
  extremities.
QUESTION No. 283
Which of the following is the most likely diagnosis?

ANSWER
a.- Still´s murmur.
b.- Atrial septal defect.
c.- Patent ductus arteriosus.
d.- Tetralogy of fallot.
CLINIC CASE
A 3 week old former full-term infant is in your office for
  her regular health maintenance visit. She is growing
  and developing of the right leg with asymmetric
  gluteal folds.

When gentle pressure is applied to the flexed and
 adducted rigth hip in a posterior direction, there is
 no detectable clunk: however, when this is followed
 by hip abduction, a palpable clunk ensure.
Preinatal history is significant for cesarean delivery
  secondary to persistent brech presentation.

The is a result of the mother´s first pregnancy.
QUESTION No. 284
Which of the following is the most likely explanation for
 these findings?

ANSWER
a.- Metatarsus adductus.
b.- Tlipes equinovarus.
c.- Developmental dysplasia of the hip.
d.- Osgood-Schlatter disease.
CLINIC CASE
A 33 year old woman who is obese (BMI 37) comes to your
   office because she heard in the news that obesity
   increases the risk for cancer.
QUESTION No. 285
You advise that patient that obesity does indeed increase
   the risk for all of the following cancer in woman of her
   age except?
ANSWER
a.- Breast cancer.
b.- Endometrial cancer.
c.- Gallbladder cancer.
d.- Renal cell cancinoma.
CLINIC CASE
In counseling a 39 year old obese BMI 34, man.

QUESTION No 286
You inform him he is at increased risk of all of the
  following except?

ANSWER
a.- Galistones.
b.- Slepp apnea.
c.- Osteoarthritis.
d.- Glaucoma.
CLINIC CASE

A 24 year old woman with history of Hashimoto´s
  thyroiditis come to a follow up visit and informs you
  that she is pregnant. Her gynecologist has started her
  on prenatal vitamins but advised her to see you
  because of her thyroid disease.
QUESTION No. 287
Which of the following is true for this patient?

ANSWER
a.- She has a low risk for developing pre-eclampsia
   during her pregnancy.
b.- Her thyroid antibodies are likely to increase during
   the pregnancy.
c.- She will need to progressively increase her dose of
   levothyroxine during her pregnancy.
d.- Her likelihood of undergoing a cesarean section is
   decreased.
CLINIC CASE

A 31 year old man with history of type 1 diabetes sice
  the age of 4 present to your office for routine follow
  up, the patient´s A1c is 7.2% and he is using an
  insulin aspart for diabetes control.
On physical exam, you find that the patient has a blood
  pressure of 142/86 mmHg. This is the first time his
  bloog pressure readings have been elevated. A spot
  urine albumin/creatinine done one month prior is
  39.
You have the patient return a week later to recheck
  blood pressure and review new laboratories. Now his
  blood pressure is 149¿6/84 mmHg and he has a spot
  urine albumin/creatinine of 36. his creatinine is 1.3
  mg/dL. It had been 1.1 mg/dL. 6 momths prior. You
  start the patient on enalapril and have him return
  two weeks later with blood chemistries.

At the next follow up visit, the patient´s blood pressure
  is now 152/88 mmHg and his creatinine is 1.7
  mg/dL, and the potassium is 5.1 mEq/L.
QUESTION No. 288
Which of the following shold be the nest step in the
 treatment of his patient?

ANSWER
a.- Order a gadolinium-enhanced MRA.
b.- Order a renal duplex ultrasound.
c.- Discontinue enalapril and start verapamil.
d.- Increse his dose of enalapril
QUESTION No. 289
Which of the following is necessary to prepare a patient
   who will be undergoing resection of a medulary
   adrenal mass which has been confirmed to be a
   pheochromocytoma?
ANSWER
a.- Liberalize salt intake(high sodium diet) at least 10 days
   prior to surgery.
b.- Discontinue antihypertensive therapy 3 days prior to
   surgery.
c.- Liberalize sal intake (high sodium diet) at least 3 days
   prior to surgery.
d.- Discontinue antihypertensive therapy 1 day prior to
   surgery.
CLINIC CASE

A 58 yer ol man presents to your office as a new
  patient. H has nt seen a physician in several years.
  The patient states that he is getting up at night
  frequently to urinate. He is thirsty most of the
  time, and he has blurry vision. He has also been
  bruising easily and has notice he looks mor tanned.
On physical examn his pulse is 82 bpm and his blood
  pressure is 166/92 mmHg. The adbomen is large
  with prple striae and extremities are thin with
  several ecchymoses. You ask the patient to squat
  and stand up but he is unable to stand up without
  assistance. The patient´s skin shows darker
  pigmentation in skin fold areas.

You order laboratories and radiological diagnostic and
  confirm a diagnosis of cushung´s disease.
QUESTION No. 290
Which of the following is recommended for this
 patient?

ANSWER
a.- Adrenalectomy.
b.- Gamma knife tratment.
c.- Ketoconazole therapy.
d.- Transphenoydal microsurgery.
QUESTION No. 291
All of the following patient have an increassed risk for
   secondary osteoporosis except?

ANSWER
a.- A 47 year old woman with history of asthma who uses
   inhaled steroids.
b.- A 38 year old women with history of epilepsy who
   requires medication to continue to suppress seizure
   activity.
c.- A 42 year old man with bipolar disorder who is on
   chronic lithium therapy.
d.- A 52 year old men with history of hypothyroidism who
   has a THS of 1.1 mIU/L.
CLINIC CASE

A 64 year old man has been diagnosed with papillary
  thyroid carcinoma. The patient underwent total
  thyroidectomy and recerved 100 mci of 1131 for
  ablation of any remaining tissue. A thyroglobulin
  level done one week after ablation is less than 1
  ng/mL. You need to start thyroid hormone as
  adjunvant therapy for the thyroid cancer.
QUESTION No. 292
What should your goal for this patient´s TSH be?

ANSWER
a.- < 0.1 uIU/mL.
b.- < 0.01 uIU/mL.
c.- < 0.4 uIU/mL.
d.- < 1.0 uIU/mL.
CASO CLINICO

Se trata de paciente femenino de 26 años de edad que
  refiere dolor desde hace 3 años el cual se ha
  incrementado paulatinamente, el dolor se presenta
  principalmente durante la menstruación, acudió a
  consulta por que ahora es incapacitante disminuye
  levemente con el uso de AINE´s y antiespasmoliticos,
  niega leucorrea, no hay datos de irritación peritoneal
  sin embargo si presenta defensa. Casada desde hace
  4 años y sin método anticonceptivo.
PREGUNTA No. 293
Considerando su impresión diagnostica, cual es su
  conducta mas adecuada?

RESPUESTA
a.- Cefotaxima.
b.- Antifungicos.
c.- Indometacina.
d.- Progestagenos
PREGUNTA No. 294
La paciente regresa 3 meses después refiriendo
  disminución significativa del dolor, con disminución
  de la cantidad de menstruación comparada con las
  anteriores, considerando la respuesta del paciente,
  cual es el diagnostico mas probable del caso?

RESPUESTA
a.- Endometriosis.
b.- Tuberculosis tubarica.
c.- Enfermedad pélvica inflamatoria.
d.- Síndrome de colon irritable.
PREGUNTA No. 295
La paciente acude a revisión refiriendo mejoría
   significativa sin embargo usted observa un
   incremento en su peso, sus pligues se encuentran
   obscurecidos, no muestra estrias violaceas ni edema
   de miembros inferiores, pero si hay acne e
   hirsutismo leve, cual es la conducta a seguir?
RESPUESTA
a.- Metformida.
b.- Analogos GnH.
c.- Antiandrogenos.
d.- Glipicida
CASO CLINICO

Se trata de femenino de 14 años de edad con tanner de
  acuerdo a edad, no cuenta con antecedentes de
  importancia, sin vida sexual activa, acudió por flujo
  vaginal anormal, por el interrogatorio hay datos que
  corresponden a episodios de opsomenorrea con
  retrasos de 30 a 50 dias, sin que ocurran síntomas
  asociados.
PREGUNTA No. 296
Considerando el cuadro clínico, cual es su conducta a
  seguir?

RESPUESTA
a.- Uso de supresión de progesterona.
b.- Uso de anticonceptivos orales combinados.
c.- Se solicita perfil ginecologico.
d.- Se solicita ultrasonograma.
CLINIC CASE

You are asked to evaluate a 32 year old men who
  presents with a sharp pain at the front of the neck
  which radiates to the jaw. The patient denies ane
  fever but does say that he is fatigued and has some
  trouble swallowing since the pain started.

He has been recovering from an influenza infection
  over the past week. Upon examination of the
  neck, there is tenderness over they thyroid qland
  with gentle pressure.
QUESTION No 297
Which of the following findings would you not expect
 to have for this patient?

ANSWER
a.- Thyroid uptake of 1%.
b.- low thyroglobulin level.
c.- fT4 level 6.4 ng/dL.
d.- TSH level 0.8 mlU/L.
CLINIC CASE

A 24 yer old woman comes to your office to ask about
  her risk for diabetes. She delivered a baby 2 months
  ago and during her pregnancy she developed
  gestational diabetes. Her glucose levels are now after
  a 2 hour glucose tolerance test.
QUESTION No. 298
You advice this patient that?

a.- She doesn´t need to be concerned as GDM is reversible
   an does no increase the risk for diabetes.
b.- She should begin a medical nutrition therapy program in
   addition to exercise since her risk for developing diabetes
   over the next 5 years is 50 %
c.- She sould begin a medical nutrition therapy program in
   addition to excercise since her risk for developing
   diabetes over the next 5 years is 25%.
d.- She should begin therapy with sitaglipin since she
   already has diabetes.
CLINIC CASE

You are evaluating a 74 year old woman who resides in
  a nursing home because of physical limitations due
  to a several strok she suffered a rear prior.
The patient has been losing weight steadily over the
  past year due to depression and poor food intake.
  The nurse who follows the patient reports that the
  patient has been having a facial twitch.
Upon examination, you are able to elicit trousseau´s
  sign. A calcium level done a week prior was 7.5
  mg/dL.
QUESTION No. 299
Which test should be ordered to verify the cause of the
  patient´s hypocalcemia?

ANSWER
a.- 1,25 (OH)2 vitamin D.
b.- PTH.
c.- 25 (OH) vitamin D.
d.- Magnesium.
CLINIC CASE

A 61 year old woman with osteoporosis come to follow up
  after she has been on alendronate for 2 years. Her BMD
  when therapy was initiated was -3.8. her new BMD
  shows her BMD to be -3.6. The pantient currently takes
  1000 Units of vitamin D and 2000 mg of calcium daily.
  Her vitamin D levels are normal and she has no histoty
  of frantures.
QUESTION No. 300
Which of the following would you recommend to this
  patient?

ANSWER
a.- Discontinue alendronate and estart teriparatide.
b.- Continue alendronate and increase vitamin D to 2000
   Units/day.
c.- Discontinue alendronate and start raloxifene.
d.- Continue alendronate and add serum calcitonin.
CLINIC CASE

A 52 year old male from monterrey reports a history
  of lyme disease and subsequent polyartheritis. He
  was treated with antibiotics 3 months ago and has
  been asymptomatic.

He went back to this primary care phymary care
  physician who retested him for lyme disease. He
  was found to have two IgM bands and five IgG
  bands (a positive test) on western blot.
QUESTION No. 301
Which of the following should he do next?

ANSWER
a.- Ignore the results.
b.- Recheck lab in 3 months.
c.- Refer for synovectomy.
d.- Doxycycline 100 mg bid for 28 days.
CLINIC CASE
A 37 year old male comes to the emergency
  department with ongoing fever upto 102 F for the
  past 4 weeks.

He has an associated diffuse nonpruritic salmon-
  colored maculopapular skin rash. He also
  complains of enlarged lymph nodes, bilateral wrist
  swelling, and muscle aches. Laboratory testing a
  negative antinuclear antibody and rheumatoid
  factor?
QUESTION No. 302
Which of the following is the most likely diagnosis?

ANSWER
a.- Becet´s diesease.
b.- Systemic lupus erythematosus.
c.- Palindromic rheumatism.
d.- Adult stull´s disease.
CLINIC CASE
You are consulted on a 72 year old female in the
  hospital for arthitis. She has a long history of
  bilateral hand paind. Her examination reveals
  subluxation of her MCP´s, ulnar deviation, and
  swan neck and boutineers deformities. There is
  significant joint tenderness of her MCPs and PIPs
  and she has splenomegaly. Her labs show a WBC
  count of 1500 and hemoglobin of 10 g/dL with a
  normal MCV. The ESR is 74 and a CRP is 16.1 mg/dl.
QUESTION No. 303
Which of the following is the most appropriate
 treatment at this time?

ANSWER
a.- Methotrexate.
b.- Splenectomy.
c.- Cyclophosphamide.
d.- Neupogen.
QUESTION No. 304
Which of the following is the less appropriate
 treatment at this time?

ANSWER
a.- Methotrexate.
b.- Splenectomy.
c.- Cyclophosphamide.
d.- Neupogen.
CASO CLINICO
Una mujer de 74 años de edad se presenta con dolor
  tipo cólico y distensión abdominal. Su H.C es
  notable por obesidad. Ella nunca ha tenido un
  episodio previo de dolor abdominal como este, pero
  sufre de ataques de dolor en cuadrante superior
  derecho después de comidas colecistocinéticas. Ella
  está afebril, normotensa, icterica, FC de 95, con
  dolor a la palpación profunda en hipocondrio
  derecho, murphy y pronn. USG con VB de 10
  cm, pared de 4 mm, Colédoco de 12 mm.
PREGUNTA No. 305

Cual es el tratamiento mas adecuado al ingreso del
  paciente?

a.- Colecistectomía laparoscópica
b.- Realizar CPRE
c.- Analgésico, antibiótico y rehidratación
 d.- Solicita PFH, QS, BH, tiempos de coagulación.
CASO CLINICO
Una mujer de 50 años se presenta con una historia de
   cuatro semanas de evolución, con dolor cólico en
   cuadrante superior derecho que se irradia a
   omoplato derecho, se encuentra en posición en
   gatillo, se observa levemente agitada, con fiebre de
   37.9 C, su IMC 36, Colesterol 433, trigliceridos
   524, bilirrubinas se encuentra normales, Se
   encuentra con mucosas deshidratadas, no hay datos
   de irritación peritoneal, niega antecedentes de dolor
   posterior a colecistoquinetico, se realizo un USG que
   no demuestra evidencia de colelitiasis o de
   coledocolitiasis.
PREGUNTA No. 306
¿Considerando los antecedentes cual es la conducta
   mas apropiada a seguir?

RESPUESTA
a.- Morfina, ceftriazona y solución mixta.
b.- Preparar para realizar CPRE.
c.- Laboratorios QS, EGO, BH, TPT y PFH.
d.- Antiespasmodico, hidratación, antibiótico y
     laboratorios.
CASO CLINICO

Se trata de un paciente masculino de 38
  años el cual acude al servicio de urgencias
  por      opresión     en      tórax,    no
  transitivo,       taquicardia,        dolor
  abdominal, refiere encontrarse bajo
  mucha presión laboral.
PREGUNTA No. 307
Tomando los datos obtenido cual es su conducta a
  seguir?

a.- Radiografia de tórax, ECG y enzimas.
b.- Oxigeno, analgesico y laboratorios
c.- Ansiolitico, sedante e hipnotico.
d.- Oxigeno, analgesico y ansiolitico
CASO CLINICO

Se trata de masculino de 51 años de edad el cual acude
  por perdida de peso de 15 kilos en 3
  meses, acompañado de dolor epigástrico continuo, el
  paciente tiene antecedentes de tabaquismo y
  alcoholismo crónico, así como gastritis y colitis de
  larga evolución, con múltiples tratamiento con
  moderado efecto. Sus laboratorios presentaron
  bilirrubina total de 21, plaquetopenia, anemia aguda
  y PFH elevadas.
PREGUNTA No. 308
Considerando su diagnostico actual, cual es la tipo
  mas probable en este caso?

RESPUESTA
a.- Insulinoma.
b.- Gastrinoma.
c.- VIPoma.
d.- Somastostinoma.
PREGUNTA No. 309
Considerando su diagnostico, con la clasificación de
  este grupo de patología que tipo es?

RESPUESTA
a.- Primitivos.
b.- Primarios.
c.- Secundarios
d.- Metastasicos.
PREGUNTA No. 310
Considerando la pregunta anterior cual es el subtipo
  mas probable?

RESPUESTA
a.- Epiteliales sólidos.
b.- Epiteliales mixtos.
c.- Epiteliales quístico.
d.- Epiteliales musinoso.
PREGUNTA No. 311
Considerando el diagnostico establecido, cual de los
  siguientes localizaciones en la segunda mas
  frecuente?

RESPUESTA
a.- Colon
b.- Pancreas
c.- Gastrico.
d.- Esofagico.
PREGUNTA No. 312
Cual de los siguientes tumores es tiene tratamiento
  paliativo únicamente?

RESPUESTA
a.- Colon
b.- Pancreas
c.- Gastrico.
d.- Esofagico.
PREGUNTA No. 313
Cual de las siguientes estructuras es la principal que
  tienen origen de cáncer de páncreas.

RESPUESTA
a.- Ductos
b.- Islotes.
c.- Periampular.
d.- Acinos
PREGUNTA No. 314
Señale cuál es la técnica quirúrgica indicada, en un
  paciente con adenocarcinoma ductal de páncreas,
  situado en la cabeza pancreática y potencialmente
  resecable:

a.- Duodenopancreatectomía cefálica.
b.- Pancreatoyeyunostomía lateral.
c.- Resección corporo-caudal del páncreas.
d.- Gastroyeyunostomía.
PREGUNTA No. 315
Ante la palpación de una vesícula distendida e indolora
  en un paciente ictérico y con síndrome constitucional
  asociado, ¿qué diagnóstico, de los siguientes, hay
  que sospechar en primer lugar?:

a.- Carcinoma vesicular.
b.- Coledocolitiasis.
c.- Cólico hepático simple.
d.- Carcinoma de cabeza de páncreas
PREGUNTA No. 316
Señale lo correcto respecto al carcinoma de páncreas:

a.- Afecta preferentemente a mujeres.
b.- Responde a la radioterapia en un 30-40% de los
   casos.
c.- Es muy quimiosensible.
d.- Cuando presenta dolor es habitualmente
   inoperable.
PREGUNTA No. 317
La duodenopancreatectomía puede estar indicada en
  los siguientes procesos, EXCEPTO en uno. Señálelo:

a.- Tumores de ampolla de Vater.
b.- Tumores de la cabeza del páncreas.
c.- Pancreatitis crónica localizada en la cabeza del
   páncreas.
d.- Tumores primitivos del retroperitoneo.
PREGUNTA No. 318
Un enfermo con ictericia progresiva, indolora
   (bilirrubina total 8 mg/dl) es diagnosticado de masa
   periampular con PAAF positiva para células malignas.
   El estudio de extensión es negativo. ¿Cuál debe ser la
   opción terapéutica más aconsejable de las
   siguientes?
a.- Drenaje externo, nutrición parenteral y continuar el
   estudio.
b.- Instalación de prótesis expansora transtumoral.
c.- Duodenopancreatectomía total.
d.- Duodenopancreatectomía cefálica
PREGUNTA No. 319
¿Cuál de las siguientes medidas preoperatorias es la
  más importante a realizar en un paciente
  severamente ictérico, por cáncer de páncreas
  posiblemente resecable, en vistas a una mejor
  evolución postoperatoria?

RESPUESTA
a.- Descompresión biliar preoperatoria.
b.- Nutrición adecuada preoperatoria.
c.- Tratamiento antibiótico preoperatorio.
d.- Profilaxis antitrombótica preoperatoria.
PREGUNTA No. 320

Los insulomas?

a.- Casi siempre requieren muestreo venoso selectivo
   para su localización.
b.- Son más frecuentes en la cabeza del páncreas.
c.- Por lo general son benignos.
d.- Se tratan con pancreatectomía anatómica.
PREGUNTA No. 321
¿ Cual de los siguientes es el síntoma de presentación
  más      frecuente     en      pacientes   con    un
  somatostatinoma?

a.- Colelitiasis.
b.- Estreñimiento.
c.- Hipoglucemia.
d.- Hipocalcemia.
PREGUNTA No. 322
¿ Cuál de los siguientes estudios de imagen es el
  más exacto para identificar tumores endócrinos
  del páncreas?

a.- Ecografía
b.- T.A.C.
c.- RM
d.- Estudio con metayodobencilguanina
PREGUNTA No. 323

¿Cuál de los siguientes es el tumor endócrino
  pancreático más frecuente?

a.- Gastrinoma.
b.- Glucogonoma.
c.- Insulinoma.
d.- Tumor productor de PIV.
PREGUNTA No. 324
¿Cuál de los hallazgos siguientes en la exploración
  física de un paciente diabético representa una
  posibilidad de Glucoganoma?

a.- Acantosis nigricans.
b.- Dematitis.
c.- Manchas café con leche.
d.- Efélides axilares.
PREGUNTA No. 325

Los síntomas y signos de un paciente con un tumor
  secretor de polipéptido intestinal vasoactivo (VIP)
  incluyen todos los siguientes, EXCEPTO:

a.- Diarrea que no responde a agentes antidiarreicos.
b.- Diarrea que persiste durante el ayuno.
c.- Hipopotasemia.
d.- Alcalosis metabólica grave.
CASO CLINICO

Femenino 28 años. Acude por historia de palpitaciones
  y temblor fino en las manos. Tiene insomnio pero lo
  atribuye al nacimiento de su bebé hace 5 semanas.
  Se encuentra lactando. A la EF tiene temblor fino
  distal, sudoración en palmas de las manos y se
  aprecia un discreto bocio.
  Labs: TSH <0.002 (0.8-1.8).
PREGUNTA No. 326
Cuál de las siguientes medidas está indicada para
  establecer el diagnóstico?

RESPUESTA
a.- Determinación de anticuerpos anti receptor TSH.
b.- Administrar I131.
c.- VSG.
d.- Medir Tiroglobulina.
PREGUNTA No. 327
La medida que usted ha elegido le permitirá
  diferenciar entre:

RESPUESTA
a.- Tiroidis viral vs. tiroiditis postparto.
b.- Tiroiditis postparto vs. enfermedad de graves.
c.- Trastorno inmunológico vs. enfermedad de graves.
d.- Tiroidistis de hashimoto vs enfermedad de graves.
PREGUNTA No. 328
Los principales marcadores de Enfermedad de Graves
  son:

RESPUESTA
a.- TSI.
b.- Tiroglobulina.
c.- Anticuerpos anti TPO.
d.- Anticuerpos anti Tiroglubilina.
PREGUNTA No. 329
La triada de la enfermedad de graves es:

RESPUESTA
a.- Oftalmopatía, Dermopatía, Osteopatía
b.- Oftalmopatía, Dermopatía, Neuropatía
c.- Hipertiroidismo, Oftalmopatía, dermopatía
d.- Hipotiroidismo, Oftalmopatía, dermopatía.
PREGUNTA No. 330
Todas son afirmaciones ciertas acerca del I 131 excepto:


RESPUESTA
a.- El embarazo y la lactancia son contraindicaciones
    para su uso
b.- Es una terapia indicada en la Enfermedad de Graves
c.- Puede generar hipotiroidismo
d.- Genera efectos de forma inmediata.
PREGUNTA No. 331
Una determinación baja de T4 libre y TSH elevada
   hacen diagnóstico de?

RESPUESTA
a.- Hipertiroidismo secundario
b.- Hipertiroidismo primario
c.- Hipotiroidismo primario
d.- Hipotiroidimo secundario
CASO CLINICO

Femenino de 19 años de edad el cual acude a
  consulta presentando dolor en epigastrio, tipo
  punzante, refiere la madre presentar nausea
  moderada y falta de apetito, niega vida sexual
  activa.
PREGUNTA No. 332
Cual de las siguientes posibilidades diagnosticas es la
  mas probable considerando su cuadro clínico. ?

RESPUESTA
a.- Embarazo Ectópico.
b.- Apendicitis.
c.- Dismenorrea.
d.- Gastritis aguda
PREGUNTA No. 333
Tomando en cuenta su diagnostico, cual es el
  tratamiento de inicio que indica?

RESPUESTA
a.- Analgésicos y antiespasmodicos.
b.- Inhibidores de la bomba de protones.
c.- Hidratación y vigilancia.
d.- Laparotomía Exploratoria.
CASO CLINICO

Un paciente de 16 años femenino que acudio a su
  unidad de medicina familiar refiriendo que 10
  horas antes inicio con dolor ubicado en la fosa
  iliaca derecha acompañado de nausea y diarrea
  leve, niega fiebre, el dolor era intenso pero
  súbitamente disminuyo, tomo medicamento sin
  indicación medica.
PREGUNTA No. 334
Cual es su conducta a seguir mas apropiada para
  este caso?

RESPUESTA
a.- Alta con cita abierta.
b.- Antibiotico y antiespasmodico.
c.- QS, BH, RX de abdomen inmediatos.
d.- Solicita HGC en sangre.
PREGUNTA No. 335
Esta patología se caracteriza por incrementar la
  presión de su luz intestinal, ya que su capacidad
  no es mayor a .01 ml. Tomando en cuenta la edad
  de la paciente su anatomía y a la probable
  etiología es cual es el agente causal?

a)   E. Coli.
b)   Sf Fecalis.
c)   B. Fragilis.
d)   E. Histolitica.
PREGUNTA No. 336
La paciente ingresada en espera de resultado se observa
  palida, diaforetica, hipotensa e hipotermica, se
  observaron leucocitosis de 25,000 considerando su
  conducta terapéutica y diagnostica cual es la conducta
  mas adecuada a seguir ahora?

RESPUESTA
a.- Tomar tiempos de coagulación
b.- Solicitar plasma fresco congelado y paquetes
     eritrocitarios.
c.- Doble esquema de antibiótico.
d.- Laparotomía exploratoria de urgencia.
PREGUNTA No. 337
Cual es la complicación crónica mas frecuente
  debido a la evolución del caso?

RESPUESTA
a.- Síndrome de mala absorción.
b.- Bridas y adherencias intestinales
c.- Trastornos motores de colon
d.- Síndrome de intestino irritable.
CASO CLINICO

Un paciente de 45 años de edad es diagnosticado
  de esófago de Barret largo (segmento de 6 cm) y
  sigue tratamiento con inhibidores de la bomba
  de protones (IBP) a dosis estándar.
PREGUNTA No. 338
Cuál de las siguientes afirmaciones es cierta?

RESPUESTA
a.- En su seguimiento se realizará endoscopia sólo si persisten
    los síntomas.
b.- Mientras se mantenga el tratamiento continuo con IBP se
    elimina el riesgo de cáncer.
c.- Se debe asegurar un control adecuado de la secreción ácida
    con dosis altas de IBP o preferiblemente ajustando la dosis
    mediante pHmetría.
d.- El seguimiento sólo es necesario si existe displasia de alto
    grado.
PREGUNTA No. 339
Tras realizar una endoscopia digestiva a un paciente de 51
   años, se nos informa que el diagnóstico de sospecha es
   "esófago de Barrett". ¿Cuál es, entre las siguientes, la
   conducta más adecuada?
RESPUESTA
a.- El paciente debe ser remitido al servicio de Cirugía
   para realizar un funduplicatura.
b.- La indicación correcta es la esofaguectomía.
c.- Lo más adecuado, para tener seguridad diagnóstica
   completa, es esperar al resultado de la biopsia antes de
   tomar decisiones.
d.- Debemos pautar tratamiento médico con omeprazol
   de por vida y no aconsejar más revisiones.
PREGUNTA No 340
En cuál de las siguientes neoplasias gástricas en
estadio incipiente se puede considerar como primera
opción la erradicación de H. Pylori con antibioterapia e
IBP?

RESPUESTA
a.- Adenocarcinoma gástrico difuso.
b.- Adenocarcinoma gástrico de tipo intestinal.
c.- Linfoma MALT gástrico.
d.- Linfoma alto grado gástrico.
PREGUNTA No. 341
Cuál de los siguientes factores NO se considera factor
  de riesgo de desarrollo del cáncer gástrico?

RESPUESTA
a.- Anemia microcítica hipocromica.
b.- Gastritis crónica atrófica.
c.- Infección por Helicobacter Pylori.
d.- Enfermedad de Ménétrier.
PREGUNTA No. 342
Señalar en relación con el linfoma gástrico primario, cuál es la
respuesta acertada?
RESPUESTA
a.-.- El linfoma gástrico primario es de peor pronóstico que el
adenocarcinoma.
b.- Los pacientes que responden a la antibioterapia en linfoma
MALT deben controlarse mediante endoscopia periódica, debido
al desconocimiento de si se elimina el clon neoplásico ó
simplemente se suprime de forma temporal.
c.- La resección gástrica asociada a quimioterapia permite
supervivencias de 80 a 90% a los cinco años en pacientes con
linfoma de alto grado localizados.
d.- El linfoma gástrico primario es una forma infrecuente de
presentación extraganglionar de linfoma.
PREGUNTA No. 343
En un paciente de 66 años, diabético no insulinodependiente, al
    que se le ha realizado una endoscopia, se ha detectado una
    lesión extensa en el cuerpo gástrico. La biopsia informa
    adenocarcinoma de tipo intestinal. ¿Cuál de las siguientes
    afirmaciones es más correcta?
RESPUESTA
a.- Unos valores de CEA, CA 19.9 y CA 72.4 en el rango de la
    normalidad nos excluyen la presencia de enfermedad
    diseminada.
b.- Es obligatorio realizar un tránsito gastrointestinal para verificar
    la extensión
c.- El siguiente paso es la realización de ecografía endoscopica y TAC
    abdominal pélvico.
d.- Solicitaremos una ecografía endoscópica para descartar la
    presencia de metástasis hepáticas y pancreáticas.
PREGUNTA No. 344
De entre las siguientes afirmaciones sobre el carcinoma
  gástrico precoz, señale cuál es FALSA:

a.- Operado tiene buen pronóstico, con supervivencia
   hasta del 95% de los casos a los cinco años.
b.- No sobrepasa la submucosa.
c.- No tiene capacidad para producir metástasis
   ganglionares linfáticas.
d.- Existe una clara correlación entre la profundidad de
   la invasión del tumor y la tasa de supervivencia.
PREGUNTA No. 345
Cuál de las siguientes entidades NO se asocia con
  mayor riesgo de cáncer gástrico que el de la
  población general?

RESPUESTA
a.- Reflujo duodenogástrico.
b.- Enfermedad de Menetrier.
c.- Anemia perniciosa.
d.- Acalasia.
PREGUNTA No. 346
El estudio de mayor especificidad y sensibilidad para
   hacer el diagnóstico de reflujo gastroesofágico es
   la:

RESPUESTA
a.- Endoscopia.
b.- Manometría.
c.- Medición del ph.
d.- Serie esofago-gastroduodenal.
PREGUNTA No. 347
Paciente de 68 años que consulta por pirosis.
  disfágía, leve ocasional y episodios compatibles con
  regurgitación nocturna hace 2 semanas. Refiere desde
  hace unos 3 años temporadas anteriores de pirosis y
  regurgitación. La endoscopia alta practicada
  demuestra una esofagitis erosiva grave. ¿Qué
  tratamiento farmacologico, entre los siguientes, es el
  más adecuado?

RESPUESTA
a.- Inhibidores de la bomba de protones.
b.- Antagonistas de los receptores H2.
c.- Sucralfato.
d.- Tratamiento combinado con anti-H2 y sucralfato.
PREGUNTA No. 347
Varón de 42 años, obeso reciente, no fumador, con asma
  desde hace 1 año, que empeora después de las comidas
  y se despierta por las noches con tos irritativa. Sigue
  tratamiento con broncodilatadores y prednisona. ¿Cuál
  sería la mejor prueba diagnóstica para aclarar el cuadro?

RESPUESTA
a.- Endoscopia bronquial.
b.- Pruebas de provocación con alergenos.
c.- pH esofágico durante 24 horas.
d.- Ecodoppler de venas de piernas.
PREGUNTA No. 348
¿Cuál de las siguientes medidas NO forma parte del
  primer paso del tratamiento del reflujo
  gastroesofágico?

RESPUESTA
a.- Elevar la cabecera de la cama.
b.- Evitar el tabaco y el alcohol.
c.- Reducir las grasas de la dieta.
d.- Administrar ranitidina, 150 mg cada 12 horas.
PREGUNTA No. 349
En un paciente con esofagitis secundaria a reflujo
  gastroesofágico, ¿cuál     de   los   siguientes
  medicamentos deberán evitarse?

RESPUESTA
a.- Teofilinas.
b.- Cisapride.
c.- Domperidona.
d.- Furosemida.
PREGUNTA No. 350
Un paciente de 32 años exhibe una historia de disfagia, de
  varios años de evolución, que ha progresado en las
  últimas semanas hasta tener dificultades, no
  diarias, para ingerir líquidos. Describe también
  episodios aislados de regurgitación no acida de
  alimentos y ha tenido, el año pasado, dos episodios de
  neumonía. La exploración que más probablemente haga
  el diagnóstico correcto será:

RESPUESTA
a.- Endoscopia digestiva alta.
b.- Radiología esofagogástrica con bario.
c.- Estudio de vaciamiento esofágico con radioisótopos.
d.- Manometría esofágica.
RESPUESTA No. 351
El reflujo gastroesofágico puede relacionarse con
  cualquiera de las entidades siguientes EXCEPTO una.
  Señálela:

RESPUESTA
a.- Asma por reflujo
b.- Broncoespasmo.
c.- Neumonía recurrente.
d.- Sinusitis.
PREGUNTA No. 352
Un paciente de 42 años de edad consulta por molestias faríngeas y
   cierta afonía de dos meses de evolución. NO es fumador ,ni tiene
   una profesión en que deba forzar la voz. Una exploración laríngea
   revela un ligero edema delas cuerdas vocales y un ligero eritema de
   la región interaritenoidea. Interrogado el paciente no refiere pirosis
   ni regurgitación ácida. ¿Cuál de las siguientes afirmaciones es
   cierta?:
RESPUESTA
a.-La presencia de mínimas lesiones laríngeas indican que muy
   probablemente el paciente tenga también lesiones de esofagitis y
   por tanto hay que indicar una endoscopia digestiva alta.
b.-En ausencia de síntomas de broncoaspiración (tos y sibilancias), las
   molestias laríngeas no pueden ser atribuidas a enfermedad por
   reflujo gastroesofágico.
c.- La ausencia de síntomas de reflujo (pirosis y regurgitación)no
   descarta la enfermedad por reflujo.
d.-Puede averiguarse si la causa de los síntomas es una enfermedad
   por reflujo gastroesofágico con una prueba corta administrando
   ranitidina 150 mg al día durante dos semanas.
PREGUNTA No. 353
El objetivo de la cirugía antirreflujo en los pacientes
   que presentan enfermedad por reflujo
   gastroesofágico es?

RESPUESTA
a.- Mejorar el vaciamiento esofágico.
b.- Restablecer la función del esfínter esofágico
   superior.
c.- Mejorar la contractilidad esofágica.
d.- Mejorar la función mecanica del cardias.
CLINIC CASE
Female patient is 67 years old, who comes in for
  tiredness, fatigue, shortness of breath, pain toraccic
  decreases with rest, dyspnea on moderate
  exertion, edema of the lower 2/3 of the leg, the
  physical examination, pallor of skin and mucous
  membranes, delayed capillary refill, mild rales, gallop
  and trill presence. The patient is hypertensive for 20
  years with moderate adherence to treatment with
  captopril 50 mg / day, hydrochlorothiazide 25 mg/12
  hrs. He has presented two hypertensive crisis, as well
  as an event of mesenteric insufficiency, laboratories
  EGO proteinuria, glucosuria, urate, decreased urine
  density, cholesterol 289 mg / dl, triglycerides 720 mg /
  dl, creatinine 6.2 mg / dl, BUM 29, kalemia of 5.3, vital
  signs were BP 160/105 mmHg, HR 96, FR 28, BMI 32.
QUESTION No. 354
Echocardiography has noted a FE <40% during
  admission, the patient presented urinary volume of
  20 ml / hr. Considering the current characteristics
  which functional class is presenting the case?

ANSWER
a. - Functional Class I.
b. - Functional Class II.
c. - Functional Class III.
d. - Functional Class IV.
QUESTION No. 355
Echocardiogram reporting an EF> 45%, premature
  ventricular electrocardiogram reports, conduct which
  is the most appropriate drug to decrease ventricular
  remodeling and improve cardiac electrical activity?

ANSWER
a. - Sign beta blocker.
b. - Start verapamil.
c. - Start losartan.
d. - Start diltiazem.
QUESTION No. 356
Considering the current characteristics which is the K/
  DOQI presented by the patient, which is the most
  suitable method for determining the stage of CKD?

ANSWER
a. - Creatinine clearance of 24 hrs.
b. - Renal biopsy.
c. - Renal ultrasonography.
d. - Escretora urography.
QUESTION No. 357
What is the most appropriate pharmacological behavior
 considering the K / DOQI and functional class?

ANSWER
a. - Verapamil.
b. - Diltiazem.
c. - Telmisartan.
d. - Isosorbide.
CLINIC CASE
Male is 59 years old, admitted to the emergency department
   because of shortness of breath, tiredness, fatigue, productive
   cough for 15 days, the night before the current admission, has a
   history of COPD diagnosed 10 years ago, after smoking (a pack a
   day), with treatment iprapropio bromide, salbutamol and
   betamethasone, and was diagnosed as hypertensive two years
   ago, starting with lifestyle changes and sodium-restricted
   diet, consider a year ago it was changed to include captopril
   three months later by cough increased, from entoces take
   losartan, hydralazine. The EF is observed with conjunctival
   hyperemia plethoric facies, moderately cyanotic oral mucosa, a
   murmur is heard right carotid, jugular venous distension grade
   II, rales subcrepitantes bibasilar, tipanica resonance to
   percussion,          slightly         diminished           heart
   sounds, rhythmic, abdomen with hepatomegaly 5 cm below the
   costal margin.
QUESTION No. 358
Considering the current state of the patient which is
  the most appropriate behavior to follow to establish
  cardiopulmonary function?

ANSWER
a. - Spirometry.
b. - Echocardiogram.
c. - Value of natriuretic.
d. - Chest radiograph.
QUESTION No. 359
After previous testing had an EF less than 40%, the chest
   radiograph showed an index cardiotoraccico pathological
   bilateral cotton wool spots, increased brocovascular
   frame, gas PCO 49, PO2 85, potassium 6.3, natriuretic factor
   decreased to physical expxloracion crackles are
   heard,     wheezing        moderate     hyperresonance    to
   percussion, galloping apresiar is reached, which is the most
   appropriate     behavior      to   continue    to   decrease
   preload, considering all factors?
ANSWER
a. - Remove potassium sparing diuretic loop diuretic.
b. - Start with dihydropyridine calcium antagonist quick action.
c. - Increased peripheral vascular capacitance nitrites.
d. - Water restriction, sodium-free diet, loop diuretic.
CLINIC CASE
Male is 68 years old which is diagnosed and treated for
  typical angina 5 years, however 6 months from the
  date of the chest discomfort was moderate and
  sometimes at rest, comes to emergency because
  oppression has accompanied toracicca left shoulder
  pain that radiates to the arm, which was added when
  he began discussing the inconvenience, did not yield to
  rest and hard over the usual time, has a history of high
  blood pressure from 20 years with current treatment
  with captopril 25 mg every 12 hrs, hydralazine 25 mg /
  12 hrs, 5 years ago was added propranolol, pravastatin
  and aspirin to manage your initially stable angina, your
  income is observed diaphoretic, with paleness
  , anxious, with distal cyanosis, his vital signs were BP
  105/70 mmHg, HR 101 bpm, RR 29 rpm.
QUESTION No. 360
Considering the clinical diagnosis which is more likely at
  this point?

ANSWER
a. - Decompensated typical angina.
b. - Initial atypical angina.
c. - Acute myocardial infarction.
d. - Angina heart failure.
QUESTION No. 361
Which of the following is most useful diagnostic aids
 at this time to maintain an adequate clinical
 behavior?

ANSWER
a. - Search biomarkers of ischemia.
b. - Perform electrocardiogram.
c. - Coronary angiography.
d. - Programming dobutamine stress test.
QUESTION No. 362
What is your class with the observed clinical
 functional?

ANSWER
a. - Class I.
b. - Class II.
c. - Class III.
c. - Class IV
QUESTION No. 363
The following studies were performed to the patient
  what is your opinion on the X-ray and ECG?

ANSWER
a. - Facts pulmonary edema.
b. - Facts heart failure.
c. - AMI data.
d. - Facts COPD.
QUESTION No. 364
What is the most important immediate behavior to
 continue?

ANSWER
a. - Morphine, oxygen, nitrite and antiplatelets.
b. - Diuretic, morphine, verapamil and oxygen.
c. - Diuretic, antiaginoso and thrombolysis.
d. - Morphine, nitrite, thrombolysis, enoxaparin.
CLINIC CASE

Turn to the outpatient female patient aged 25, comes
  because it does not produce enough milk to
  breastfeed has 8 days postpartum, denies other
  symptoms, he explains the technique for
  lactation, however the observed increase exploration
  symmetrical thyroid gland not palpable nodules or
  painful areas, not identify voice changes, the patient
  is originally from Chiapas.
QUESTION No. 365
What the most appropriate action to take in case?

ANSWER
a. - Conduct assessment of HRT and T4.
b. - Indicate iodine supplementation.
c. - Indicate diet rich in iodine, cites six months.
d. - Request antithyroid antibody profile.
QUESTION No. 366
6 months later was sent to endocrinology because it
  showed increased weight remained the same
  characteristics of volume and consistency of the
  thyroid gland, thyroid profile was made ​with the
  following results low total T4, normal TSH and
  indicated 100 mcg / day considering the
  evolution, which of the following differential diagnosis
  should nonthyroidal causes.

ANSWER
a. - Low T4 syndrome.
b. - Nontoxic multinodular goiter.
c. - Toxic multinodular goiter.
d. - Hyperfunctioning nodule.
CLINIC CASE

Subclinical hypothyroidism patient diagnosed and
  treated properly with oral thyroxine. A few months
  later, he developed bilateral exophthalmos and
  simultaneously presents serum antibodies anti-TSH
  receptor positive. Then adjourned thyroxine
  treatment, and it is found that hypothyroidism has
  progressed to become primary. A diagnosis of
  hypothyroid Graves' disease.
QUESTION No. 367
What is the most frequent complication in the long
 term for the patient due to treatment should be
 instituted.

ANSWER
a. - Osteoporosis.
b. - Arteriosclerosis.
c. - Pulmonary edema.
d. - Dyslipidemia.
CLINIC CASE

Attend annual review consultation female patient of 31
  years, which has the diagnosis of simple
  goiter, presented auscencia refers to menstrual
  periods on 4 occasions in the year, in addition to
  presenting upper airway infections on two
  occasions, has presented fatigue, generalized
  weakness, weight gain. Physical examination shows
  bradypsychia and bradilalica, looks dry and rough.
  Vital signs were BP 110/70 mmHg.
QUESTION No. 368
The patient was admitted because of malaise and
  hypothermia, which is the immediate conduct to
  follow to establish a presumptive diagnosis.

ANSWER
a. - BH, QS, EGO.
b. - Request TSH, T4.
c. - Apply thyroid peroxidase antibodies.
d. - Thyroid biopsy.
QUESTION No. 369
The following results were obtained hematocrit
  32%, hemoglobin 10.1, WBC 5,900, 219 Central
  glucose, cholesterol 310. Considering the results
  which is the most appropriate action to take.

ANSWER
a. - Start levothyroxine 150 mcg / day.
b. - Apply thyroid antibody profile.
c. - Stabilization and shipping back to the second level.
d. - Administration and prednisone 20 mg levothyroxine
   100 mcg / day.
QUESTION No. 370
2 months after the patient returns to the first level
  control of attention in its counter-note is the
  diagnosis of Hashimoto's thyroiditis, considering the
  evolution of the patient, which of the following is
  least likely comorbidities.

ANSWER
a. - Diabetes mellitus.
b. - Addison's Disease.
c. - Pernicious anemia.
d. - Pituitary leukocyte infiltration.
CLINIC CASE
Girl, 12 years old, who for 4-5 months started with
  nervousness, tachycardia, heat intolerance, sweating
  and impulsivity to eat, no weight loss or insomnia. Also
  refers bulge in the front of the neck, without pain or
  other signs of inflammation associated. His mother has
  hypothyroidism treated with L-thyroxine for 7 years. In
  Physical examination revealed bilateral ocular
  protrusion and a grade II goiter, elastic
  consistency, without pain or other signs of
  inflammation. The study showed hyperthyroidism with
  thyroid hormones elevated thyroid antibodies. Thyroid
  ultrasonography objectified both lobes enlarged (right
  18 x 18.8 x 53mm, left 15.9 x 18 x
  51mm), heterogeneous, hypoechoic areas with
  multiple indistinct borders that shaped and marked
  with Doppler hypervascularity.
QUESTION No. 371
What is the most appropriate therapeutic measure
 long term?

ANSWER
a. - Partial thyroidectomy.
b. - Radioactive iodine.
c. - Propylthiouracil.
d. - Methimazole.
CLINIC CASE
This male patient of 60 minutes RN, obtained by
  caesarean iteratively presenting other fetal
  distress, by intrauterine activity decreased and FC
  decreases up to 110 bpm is observed with very fine
  movement at the beginning and shortly presented
  mainly mild cyanosis periungeal and oral, oral
  secretions are abundant apresiaron, physical
  examination normal bilateral rales are heard, little
  audible peristalsis and temperature of 36.1
  degree, apresió tinted secretions expelled meconium
  during scanning.
QUESTION No. 372
Considering the clinical picture what their conduct to
  follow?

ANSWER
a. - Keep the patient under observation continuously.
b. - Search psobles hidden pathologies.
c. - Perform chest radiography.
d. - No pathological data are adaptive.
QUESTION No. 373
What is the therapeutic less appropriate at this time
 of the case?

ANSWER
a. - Hold fast to identify cause.
b. - Place in neutral environment.
c. - Administer oxygen by headbox 40%.
d. – Gas analysis and pulse oximetry.
QUESTION No. 374
Which of the following is not normal blood gas
 parameters in the case expect?

ANSWER
a. - pH <7.34.
b. - PaCO> 45 mmHg.
c. - HCO3> 40mEq / L.
d. - SpO2 <95.
CASO CLINICO
Se ingresa a paciente masculino de 45 años de edad
  el cual cuenta con antecedente de ser portador
  de VIH positivo con cifras de CD4 de 85,
  actualmente        suspendió     el    tratamiento
  antirretroviral, el motivo del ingreso es debido a
  dolor en la región lumbar intenso el cual mostro
  datos de ostemielitis, el paciente cuenta con el
  antecedente de presencia de tos productiva con
  esputo amarillento y sanguinolento, perdida de
  peso importante, por lo que previamente se
  realizo baciloscopia siendo positiva para
  tuberculosis, apoyada por los parámetro
  radiográficos.
PREGUNTA No. 375
Cual es el tratamiento inicial mas adecuado.

RESPUESTA
a.- Isoniazida, rifampicina, etambutol y pirazinamida.
b.- Isoniazida, rifampicina, etambutol y pirazinamida
   mas TRV.
c.- Isoniazida, rifampicina, etambutol, pirazinamida y
   estreptomicina.
d.- Isoniazida, rifampicina y etambutol.
CASO CLINICO
Paciente de 56 años, sexo femenino, con antecedente de
  dermatomiositis, en tratamiento con hidroxicloroquina
  200 mg, metotrexato 10 mg y prednisona 30 mg diarios.
  Fue hospitalizada por cuadro de 10 días de evolución, con
  fiebre, compromiso del estado general y pérdida de peso,
  encontrándose crepitaciones bilaterales al examen
  pulmonar. La TAC de tórax mostró un infiltrado nodular
  bilateral, extenso, con cavitaciones biapicales. La
  baciloscopia obtenida por lavado bronquio-alveolar (LBA)
  resultó positiva y la reacción de polimerasa en cadena
  (RPC) positiva para Mycobacterium tuberculosis. Además
  úlceras en los pulgares aparecidas cuatro semanas previas
  al ingreso. Al examen físico se observó una úlcera de
  bordes bien delimitados con fondo sucio y bordes
  necróticos, no dolorosa, ubicada en el pulpejo del dedo
  pulgar izquierdo. S. aureus y Candidaparapsilosis. La
  tinción de Ziehl Neelsen en tejido fue positiva.
PREGUNTA No. 376
Cual es el origen mas probable que cause las ulceras en
  pulgares?

RESPUESTA
a.- Farmacologica.
b.- Bacteriana.
c.- Autoinmune.
d.- Mixta.
CASO CLINICO
Se trata de masculino de 19 años de edad ingresado al
  servicio de urgencias presentando fiebre, cefalea
  intensa, dolos abdominal difuso pero ligero, pero con
  mialgias intensas, al examen físico se observa con 38,5
  c, FC 112, TA 120/70 su estado general es adecuado en
  crecimiento y desarrollo con diaforesis, se mantiene
  alerta y orientado, se realizan estudios de laboratorio y
  gabinete de rutina donde se observa únicamente
  plaquetas con 84,000, se realiza puncion lumbar donde se
  observa auscencia de eritrocitos, proteínas y glucosa
  normales, solo se observa 5 monocitos. Como
  antecedente de importancia refiere regresar de un
  campamento         y       realizar     escalado       de
  montañas, actualmente continua en el servicio de
  medicina interna.
PREGUNTA No. 377
Considerando los hallazgos así como los antecedentes
  usted integra un diagnostico, y debe iniciar
  tratamiento, cual es el más adecuado?

RESPUESTA
a.- Rifampicina.
b.- Doxiciclina.
c.- Tetraciclina.
d.- Ceftriaxona.
PREGUNTA No. 378
Cual de las siguientes contraindicaciones son menos
  probables de la farmacoterapia elegida?

RESPUESTA
a.- Incremento de los efectos tóxicos de alcaloides
   ergotamínicos y metotrexato.
b.- Disminuir la eficacia de los anticonceptivos orales.
c.- El uso concomitante con metoxiflurano puede
   producir toxicidad renal fatal.
d.- Disminuye los efectos tóxicos de ciclosporina A.
CASO CLINICO
Ingresa paciente masculino de 48 años de
  edad, ingeniero que acaba de regresar de convención
  en medio oriente, presenta fiebre, diaforesis, ataque
  al estado generalizado, fatiga y adinamia, así como
  obnubilación, a la exploración se observa hepato y
  esplegnomegalia leve a moderada, es ingresado para
  realizar estudios encontrando paludismo por
  plasmodium falciparum, parasitemia de 7
  %, hematocrito de 21 %, bilirrubina de 7.8 mg/100
  ml, creatinina 2.6 mg/100 ml. Se ingresa
  posteriormente a cuidados intensivos, se realiza
  asistencia ventilatoria y se indica neuroproteccion
  debido a la gravedad del cuadro.
PREGUNTA No. 379
Cual es el mejor esquema de tratamiento para este
  caso o prevenirlo?

RESPUESTA
a.- Quinina intravenosa.
b.- Quinidina Intravenosa.
c.- Artesunato Intravenoso.
d.- Cloroquina.
PREGUNTA No. 380
Considerando los efectos adversos, cual de los
  siguientes casos es menos probable contraindicarlo?

RESPUESTA
a.- Paciente psicótico.
b.- Paciente cardiópata.
c.- Paciente nefropata.
d.- Paciente hepatopata.
PREGUNTA No. 381
Considerando el frotis cual de las siguientes formas se
  encuentran presentes?

RESPUESTA
a.- Trofozoitos.
b.- Eritrozoitos.
c.- Quistocitos.
d.- Gametozoitos
PREGUNTA No. 382
Cual de las siguientes especies es el huésped original
  del agente etiológico del caso?

RESPUESTA
a.- Gorilas.
b.- Humanos.
c.- Moscos.
d.- Vacas
CASO CLINICO
Se trata de paciente masculino de 57 años de edad el cual
  acude al servicio de urgencias debido a tos intensa no
  productiva y fiebre, el paciente cuenta con los
  siguientes       antecedentes,       originario      de
  guanajuato, empleado de la construcción, alcoholismo
  y     tabaquismo      positivo    desde      hace    40
  años, actualmente bajo tratamiento por EPOC y cirrosis
  hepática, se realizan estudio de laboratorio y gabinete
  de rutina donde se observa un reporte con disminución
  de la respuesta inmunológica esperada, la radiografia
  de torax presenta un patrón característico, sin embargo
  el cuadro respiratorio bajo se ha intensificado por lo
  que se traslada a terapia intensiva, donde se reporta
  positivo para legionella.
PREGUNTA No. 383
Cual es tratamiento antibiótico más apropiado para
  este caso?

RESPUESTA
a.- Azitromicina.
b.- Levofloxacina.
c.- Trimetoprim/sulfametoxazol.
d.- Penicilina.
PREGUNTA No. 384
Considerando el agente etiológico, cual de las
  siguientes fuentes de contagio no es relevante para
  adquirirlo?

RESPUESTA
a.- Sistema de aguas de grandes edificios.
b.- Sistemas de agua de hoteles y hospitales.
c.- Maquinas de rocio, spas y fuentes de agua termal.
d.- Sistemas de aire acondicionado.
PREGUNTA No. 385
Cual de los siguientes medios auxiliares no es util para
  su diagnostico?

RESPUESTA
a.- Hibridación con sondas moleculares específicas.
b.- Cultivo en medio BCYE-alpha Agar + Antibióticos.
c.- Positivo en las pruebas de la catalasa y oxidasa.
d.- Medios con sangre (Agar sangre o Agar chocolate).
CASO CLINICO
Se trata de paciente femenino de 38 años de edad la
  cual acude al servicio de urgencias debido a que
  presenta fiebre, cefalea y dolor en la región de la
  nuca, como antecedentes de importancia la paciente
  trabaja en un aviario, no es diabética ni hipertensa,
  aparentemente sana antes de este episodio, a la
  exploración física se observa conciente, irritable,
  orientada, facies algicas, se realiza TAC de cabeza el
  cual no revela datos de importancia, se realiza
  punción lumbar reportándose presión de apertura de
  20 cmH2O, leucocitos 15 células (90 % monocitos),
  proteínas de 0.6 g/L, glucosa de 50mg/dl y tinción
  positiva con tinta china.
PREGUNTA No. 386
Cual es el tratamiento mas adecuado para el caso?

RESPUESTA
a.- Anfotericina.
b.- Fluconazol.
c.- Cexfriaxona.
d.- Vancomicina.
PREGUNTA No. 387
Cual de las siguientes manifestaciones es menos
  probable en esta patología?

RESPUESTA
a.- Meningoencefalitis, con atrofia cortical.
b.- Presentan cuadros psiquiátricos.
c.- Durante muchas semanas no da síntomas.
d.- Difícilmente presentan fiebre.
PREGUNTA No. 388
Cual de los siguientes agentes diferenciales es menos
  frecuente?

RESPUESTA
a.- Histoplasmosis.
b.- Aspergilosis.
c.- Candidiasis.
d.- Toxoplamosis
PREGUNTA No. 389
Cual de las características del mecanismo de acción
  del tratamiento elegido, no se presenta?

RESPUESTA
a.- Salida de iones sodio.
b.- Salida de iones cloro.
c.- Salida de iones potasio.
d.- Salida de hidrogeniones.
PREGUNTA No. 390
Cual de las siguientes manifestaciones adversas es menos
  frecuente de observarse?
RESPUESTA
a.- Aumento de fosfatasa alcalina, aumento de
   urea, anomalías en pruebas de función hepática.
b.- Taquicardia supraventricular, bradicardia, fibrilación
   auricular, bloqueo auriculoventricular de segundo grado
   y extrasístoles ventriculares.
c.- Acidosis, alteraciones de
   potasio, calcio, cloruros, fosfato, alteraciones del
   equilibrio
   hídrico, hiperbilirrubinemia, hipopotasemia, hipomagnes
   emia.
d.- Hipertensión, hipotensión, astenia, edema generalizado.
CASO CLINICO
Se trata de masculino de 34 años de edad el cual tiene
  catéter para hemodiálisis, actualmente con renopatia
  terminal, acude a consulta debido a que presenta fiebre y
  dolor intenso en la región baja de la espalda, a la
  exploración            física          se        observa
  ansioso, irritable, diaforético así como ataque al estado
  generalizado sin embargo signos vitales 120/80, FC 89, FR
  21, temperatura 38.7 C, el sitio donde se encuentra
  colocado el catéter se observa rojo y caliente, no se
  observan secreciones ni olor, la biometría hemática
  reporta 16 700 con 13 % de bandas, se realiza IRM donde
  se reporta lesiones compatibles con osteomielitis
  vertebral.
PREGUNTA No. 391
Cual es agente etiológico más probable?

RESPUESTA
a.- Streptococcos.
b.- Stafilococcos.
c.- Mycoplasmas.
d.- Tuberculosis.
PREGUNTA No. 392
Cual de los siguientes agentes es el mas virulento?

RESPUESTA
a.- Staphylococcus aureus
b.- Staphylococcus epidermidis,
c.- Staphylococcus saprophyticus
d.- Staphylococcus haemolyticus.
PREGUNTA No. 393
Cual de los siguientes no son factores de virulencia y
  sirven además de evasores de la fagocitosis para la
  especie diagnosticada?

RESPUESTA
a.- La presencia de catalasa.
b.- La presencia de coagulasa.
c.- La fermentación del azúcar manitol.
d.- Presencia de B lactamasa.
PREGUNTA No. 394
Debido a la producción de toxinas, cual de las
  siguientes manifestaciones es menos frecuente?

RESPUESTA
a.- Diarreas, vómito, náuseas.
b.- Separa el estrato granuloso del córneo.
c.- Forúnculosis asepticos.
d.- Impétigo ampolloso.
CASO CLINICO
Femenino de 60 años la cual trabaja como intendencia en
  una veterinaria, refiere que hace 10 días inicio con
  fiebre, escalofríos, cefalea, nauseas, dolor muscular, se
  automedica con fármacos sintomáticos, mejorando su
  estado general sin embargo regresan los síntomas y
  agregandoce ictericia, a la exploración clínica se presenta
  38.9 C temperatura, FC 1010, TA 140/90 mmHg, la
  saturación por oximetro 92 %, a la percusión se presenta
  dolor hepático, se percibe leve crecimiento, sin
  esplenomegalia, los estudios de laboratorio BUN de
  64, creatinina 3.6, bilirrubina de 64, AST 86, ALT
  103, fosfatasa alcalina de 390, leucocitos 11 000, 13% de
  bandas y 80 % polimorfonucleares, hematocrito 33% y
  plaquetas de 145, punción lumbar revela pleocitosis, TAC
  de torax muestra inflitrados en flama difusos compatibles
  con hemorragia pulmonar. (ver imagen)
PREGUNTA No. 395
Cual es diagnostico más probable?

RESPUESTA
a.- Neumonitis intersticial aguda.
b.- Leucemia mieloide aguda.
c.- Fiebre por Streptobacillus moniliformis.
d.- Infección por Leptospira interrogans.
PREGUNTA No. 396
Considerando el agente etiológico, cual de los
  siguientes animales es el menos transmisor?

RESPUESTA
a.- Perros.
b.- Ratas.
c.- Gatos.
d.- Humanos.
PREGUNTA No. 397
Considerando la patogenia del padecimiento, cual de
  la siguiente descripción no es habitual?

RESPUESTA
a.- La bacteria entra al organismo por la piel o
   mucosas.
b.- Se disemina por la sangre atacando diversos
   órganos.
c.- Se replica en el hígado y riñones.
d.- Se elimina por la orina, para transmisión.
PREGUNTA No. 398
Considerando el cuadro clínico, cual de los
  siguientes diagnósticos diferenciales es menos
  frecuente?

RESPUESTA
a.- Dengue.
b.- Fiebre amarilla.
c.- Malaria.
d.- Influenza.
CASO CLINICO
Femenino de 23 años de edad, acude a consulta refiriendo
  fiebre, escalofríos así como dolor de espalda y malestar
  generalizado. Cuenta con antecedente de trastorno
  bipolar y abuso de sustancias, actualmente bajo
  tratamiento sin embargo con recaídas. A la exploración
  física se observa lesiones tipo punción en pliegue
  cutáneo en antebrazo por probable inyección intravenosa
  de sustancias, al parecer de drogas sin aceptarlo la
  paciente, los signos vitales son 38.5 C de temperatura, FC
  110 lpm, FR 24, tensión arterial de 110/60 mmHg, Rx de
  tórax con varios infiltrados nodulares periféricos con
  cavitaciones.
PREGUNTA No. 399
Cuál es la patología más grave y mas probable que
  la paciente presenta?

RESPUESTA
a.- Neumonia
b.- Endocarditis.
c.- Septicemia.
d.- CID.
CASE REPORT
These male patient aged 49 who comes to the office
  because it has a palpable mass in the mandibular
  angle, the patient has a history of major importance
  such as alcoholism (daily) for 10 years, mostly bad
  health habits mouth, overweight and glucose
  intolerant untreated, the patient reported that the
  mass which has grown slowly comes, occasionally
  pressing the drains, which presents yellowish
  discharge, exploration shows an inflamed area of 6 x
  6 cm, with presence of yellowish secretion, is soft
  and indurated to the periphery and slightly tender to
  palpation, a sample for culture.
QUESTION No. 400
What is the agent most likely considering the
 clinical case.

ANSWER
a. - Actinomyces.
b. - Mucormycosis.
c. - Nocardia.
d. - Peptostreptococcus.
CASE REPORT
These male patient aged 19 who is admitted to hospital
  because his house was found confused, disoriented
  with fever, relatives refer the patient often infectious
  types of upper airway, with no treatment usually are
  completed, a physical examination is widespread in
  poor, diaphoretic, febrile with a stiff neck, and
  equally without compromise cardiopulmonary
  abdomen, lumbar puncture is performed where
  leukocytes reports of 1800, 95% neutrophils, glucose
  35 mg / dl and 100 mg protein / dl.
QUESTION No. 401
What is the most appropriate       antibiotic
 pharmacological approach.

ANSWER
a. - Ampicillin and vancomycin.
b. - Ampicillin and gentamicin.
c. - cefotaxime and doxycycline.
d. - cefotaxime and vancomycin.
CASE REPORT
Female patient is 70 years old which she consults
  because it has for several months, nonproductive
  cough, fatigue, weight loss, the patient has a history
  of tuberculosis over 20 years ago which was treated
  be able to obtain sputum for culture as well as blood
  cultures, lab reports reported presence of acid-fast
  bacilli Mycobacterium avium complex indicating, the
  chest radiograph reports bronchiectasis and small
  nodules scattered throughout the lung parenchyma.
  The patient has no other important background for
  the clinical picture.
QUESTION No. 402
What is the best treatment scheme?

ANSWER
a. - Clarithromycin and ethambutol.
b. - Clarithromycin and rifampicin.
c. - Levofloxacin and rifampin.
d. - Prizinamida, Isoniazid, rifampicin and ethambutol.
QUESTION No. 403
Which of the following means of laboratory
 diagnosis is not useful for this entity?

ANSWER
a. - (Lowenstein-Jensen).
b. - (Truant).
c. - (Middle-brook).
d. - (Ziehl-Neelsen and Kinyoun).
CASE REPORT
This is a woman, 21 years old, who presented an acute
  febrile 39-40 ° C, followed on successive days of a
  generalized itchy rash. After your health in a pediatric
  center of its region, was diagnosed clinically and
  serologically classical dengue and treated
  symptomatically with paracetamol. A week
  later, once the acute phase symptoms were in an
  almost complete regression phase, began
  experiencing       slurred      speech      and      to
  wander, dysphagia, progressive muscle weakness in
  all four limbs and decreased level of consciousness.
QUESTION No. 404
What is the most appropriate diagnostic behavior to
 identify the cause of the complications?

ANSWER
a. - Proof of thick
b. - Peripheral smear
c. - Liver function tests.
d. - Biometia hematic.
QUESTION No. 405
Which of the following statements is most useful for
 prognosis of the case?

ANSWER
a.- Fever of sudden onset.
b.- Headache
c.- Joint pain
d.- Muscle pain
CASO CLINICO
Se trata de masculino de 54 años de edad el cual cursa con
  cirrosis alcoholica e hipertensión portal asi como varices
  esofágicas y encefalopatía hepática, se encuentra
  ingresado en medicina interna, debido a que inicio hace
  tres días con alteración del estado de alerta, a la
  exploración se encuentra con temperatura de 38.4 C, FC
  110, FR 14, TA 100/60, continua con disminución del
  estado de alerta, se observa asterixis leve, su estado
  cardiopulomnar no reporta datos agudos, el abdomen se
  encuentra distendido y tenso pero no hay
  hipersensibilidad, no hay datos de sangrado de tubo
  digestivo, la biometría hemática reporta 4,800 leucocitos,
  hematocrito de 32% plaquetas de 95 000.
PREGUNTA No 406
Cual método diagnostico es el mas adecuado para
  establecer un diagnostico?

RESPUESTA
a.- Panendoscopia.
b.- Paracentesis.
c.- Lactulosa.
d.- Concentración de amoniaco.
PREGUNTA No. 407
Cual de las enfermedades metabólicas congénitas del
  hígado que conducen a cirrosis hepática es la menos
  frecuente observar.

RESPUESTA
 a.- Hemocromatosis primaria.
b.- Enfermedad de wilson.
c.- Deficiencia de alfa-1 antitripsina.
d.- Esteatohepatitis asociada a diabetes y dislipidemia.
PREGUNTA No. 408
El paciente presenta insuficiencia renal prerrenal
  funcional, que condición del SHR es menos probable
  observar?

RESPUESTA
a.- Reversible sin que exista alteración de la estructura
   renal.
b.- Se produce debido a una intensa vasoconstricción
   renal
c.- Tipo I con pronostico fatal a corto plazo.
d.- Tipo III con forma más insidiosa, con un pronóstico
   ligeramente mejor.
PREGUNTA No. 409
Se reportaron los siguientes datos, albumina
  2.1, bilirrubina total 35, INR 12, según la
  clasificación de Child-Pugh, en que estado se
  encuentra el paciente?

RESPUESTA
a.- A.
b.- B
c.- C
d.- D
PREGUNTA No. 410
Cual es su sobrevida del caso a 2 años?

RESPUESTA
a.- 85
b.- 57
c.- 35
d.- 18
CASE REPORT

Male is 25 years dedicated to the field, reports that
 while working was bitten by strange dog, which
 failed to capture, known cases of rabies in the area
 in undomesticated animals, comes to the office an
 hour later the fact.
QUESTION No. 411
What is the most appropriate behavior to follow in this
 case.

ANSWER
a. - Wash the wound and surveillance.
b. - Wash the wound and administration of tetanus.
c. - Wash the wound, administration of tetanus immune
   globulin and implement human.
d. - Wash the wound, administer tetanus, applying
   human immune globulin and human diploid cell
   vaccine.
QUESTION No. 412
In the following structural protein which is the major
  antigenic component, responsible for the formation
  of neutralizing antibodies are those that confer
  immunity?

ANSWER
a. - Glycoprotein.
b. - Matrix proteins.
c. - Nucleoprotein.
d. - Nucleocapsid.
QUESTION No. 413
Which of the following considerations should not be taken into
  account for the use of immunoglobulins?

ANSWER
a. - Do not exceed the dose prescribed by your potential
    interference with the production of antibodies induced by
    vaccination.
b. - Not used in the same site as the vaccine, or reuse the
    syringe.
c. - ot recommended in individuals previously immunized with
    diploid cells.
d. - Gamma globulin provides immediate protection, which lasts
    about 42 days.
CASE REPORT
These male patient aged 30 who attends a program of field
  work carried out in a camp at the end of the day we made a
  cookout where they served a barbecue, the return of the
  course presents moderate discomfort beginning to
  intensify,                characterized                 by
  fever, headache, malaise, abdominal pain and diarrhea, a
  physical    examination    is    observed     only   vague
  discomfort, laboratory studies reported slightly elevated
  leukocyte and erythrocyte, Wright staining shows
  neutrophils, adds that because previously suffered from
  inflammatory bowel disease was suggested a colonoscopy
  with biopsy which was conducted where the following was
  reported, mucosal infiltration by neutrophils, monocytes
  and eosinophils, with epithelial damage and loss of mucus
  also glandular degeneration and cryptic abscesses.
QUESTION No. 414
Which of the following is the most likely pathogens in
 this case?

ANSWER
a. - Escherichia.
b. - Norwalk.
c. - Campylobacter.
d. - Salmonella.
QUESTION No. 415
Which of the following observations is less frequently
 observed agent?

ANSWER
a. - has an incubation period of 2 to 5 days.
b. - is manifested mainly by the appearance of
   fever, abdominal pain, and diarrhea.
c. - can cause reactive arthritis, Guillain-Barré
   syndrome.
d. - interpersonal contact transmission or by contact
   with infected animals.
CASE REPORT

Query is presented male age 19 which states that from
  a previous day begins clinical picture characterized
  by cramping abdominal pain and watery diarrhea
  which has increased over the course of hours, is
  currently a college student and food consumed in the
  street.
QUESTION No. 416
What is the most appropriate treatment at this time
 of suffering.

ANSWER
a.- Metronidazole: 250 mg orally 1 capsule c / 8 h for
   5 days.
b.- Tinidazole 600 mg / kg / day orally for 3 days or 2
   grams in a single dose in adults.
c.- Furozalidone: 7 mg / kg / day orally for 8 days.
d.- Albendazole: 400 mg / orally for five days.
QUESTION No. 417
Considering the epidemiology of the disease, which
  of the following observations is less frequent?

ANSWER
a. - It is a cosmopolitan parasitosis
b. - Predominant in children.
c. - They are usually asymptomatic in children.
d. - Causes malabsorption and malnutrition.
QUESTION No. 418
Which of the following observations about the current
 illness, is more useful for diagnosis?

a. - Direct observation of ovoid cysts.
b. - A negative test excludes infection.
c. - The method coproparasitoscopic by Faust.
d. - duodenal biopsy.
QUESTION No. 419
Which of the following observations about the current
 illness, is more useful for diagnosis?

a. - Direct observation of ovoid cysts.
b. - A negative test excludes infection.
c. - The method coproparasitoscopic by Faust.
d. - duodenal biopsy
QUESTION No. 420
Which of the following adverse drug reactions in the
 most frequently chosen?

ANSWER
a. - Nausea, diarrhea and a metallic taste in the mouth.
b. - Has been accompanied by thrombophlebitis.
c. -
   Headache, dizziness, vomiting, glossitis, stomatitis, p
   aresthesia or dark colored urine or reddish brown.
d. - Emergence of a black
   tongue, leukopenia, neutropenia.
CASE REPORT
Male 51 years old who starts for 3 days with productive
  cough, generalized state attack, fever, shortness of
  breath, chest film in compliant data can be seen
  bottom right lobar pneumonia and a 1.5-cm round
  nodule upper lobe left untreated presenting
  improvement graduating in a week, outpatient CT
  scan, which shows nodule of 1.5 by 1.8 located in the
  same position, no calcifications are seen with slightly
  scalloped edges, mediastinal lymphadenopathy are
  not observed or pathological data, not appreciated
  exudates.
QUESTION No. 421
What is the most appropriate study to continue the
 diagnosis in this case?

ANSWER
a. - MRI
b. - FDG PET
c. - Bronchoscopy.
d. - CT scan at 6 months.
QUESTION No. 422
In nonsmokers, the onset of this condition is the result
   of a combination of factors, which is moreless
   common in women.

ANSWER
a. – Genetic.
b. - Exposure to radon gas.
c. - Air pollution.
d. – Secondhand smoke.
CASE REPORT
Male 71 years old with back pain for two months of sudden
  onset without precipitating factor, adding that the pain
  increases and decreases when lying down during the day
  and with movement. The patient is from the State of
  Mexico, was a construction worker with alcoholism and
  smoking positive for over 30 years, which left 15 years
  ago, not diabetic or hypertensive, the physical
  examination is observed with adequate general
  appearance, alert and oriented body mass index of 25, is
  made of routine laboratories where only performed
  finding elevated alkaline phosphatase and column
  radiography lytic lesion is observed only in the L3
  vertebra.
QUESTION No. 423
The clinical picture indicates a          probable
  malignancy, which is the most likely?

ANSWER
a. - Pancreatic Cancer.
b. - Gastric cancer.
c. - Thyroid cancer.
d. - Lung cancer.
QUESTION No. 424
What is more likely to find such?

ANSWER
a. - Cell Lung Cancer Small
b. - Carcinoid
c. - Lung cancer non-small cell
d. - Unspecified Lung Cancer
QUESTION No. 425
Considering the type of which of the following
  condition is less common subtypes?

ANSWER
a. - Papillary adenocarcinoma
b. - Adenocarcinoma (not specified)
c. - Bronchioloalveolar carcinoma
d. - Adenosquamous carcinoma
CASE REPORT
Male is 73 years old which is from Merida, now lives
  alone, suffers from diabetes mellitus for 15
  years, hypertension, triglycerides, cholesterol and
  high uric acid, the questioning is unknown dosage for
  his     sufferings,      refers     indicated      that
  glibenclamide, bezafibrate, enalapril, allopurinol and
  pravastatin, refers to the family for more than two
  years have trouble remembering things he is
  doing, has become suspicious, hidden things to
  prevent the theft, poverty thought, thought and
  ideas lock damage and theft.
QUESTION No. 426
Due to multiple disease entities in the patient which
  is most likely neurological complication.

ANSWER
a. - Vascular dementia is the most likely.
b. - Alzheimer's Dementia is the most likely.
c. - Prion Dementia is the most likely.
c. - Pick's dementia is the most likely.
CASE REPORT
Male is 65 years old which comes because for four months
  has asymmetric distal weakness, and muscle spasms and
  twitches concerns that arise with voluntary
  movements, physical examination shows weakness of the
  hands and extensor and difficulty moving the tongue and
  face,    muscle stretch      reflexes   are observed
  increased, exploring the sensitivity changes are not
  observed, the mental examination is ongoing thought
  and language and appropriate content, humor and
  affection preserved, other higher mental functions are
  intact.
QUESTION No. 427
What is the most likely clinical diagnosis?

ANSWER
a. - Amyotrophic lateral sclerosis.
b. - MS.
c. - Transverse myelitis.
d. - Guillain-Barre Syndrome.
QUESTION No. 428
Which of the following differential diagnoses is
 moreless likely?

ANSWER
a. - Spinal Muscular Atrophy
b. - Primary lateral sclerosis.
c. - Hawthorn progressive bulbar muscular atrophy.
d. - Olivopontocerebellar degeneration.
QUESTION No. 429
Which of the following treatments has a function for
 reducing the symptoms reported?

ANSWER
a. - Memantine.
b. - Rivastigmine.
c. - Topiramate.
d. - Galantamine
CASE REPORT

Female is 27 years old which has 22 weeks pregnant,
  goes to his third prenatal visit, to review the records
  you see it has provided independent third time a
  blood pressure of 140/90, you perform a routine test
  to support his suspected diagnosis of pre-eclampsia,
  shows lower limb edema, proteinuria, constant
  headache.
QUESTION No. 430
Which of the measures is not appropriate?

ANSWER
a. - Outpatient management.
b. - Hydralazine 50 mg every 12 hrs.
c. - hiposodia diet and fluid restriction.
d. - Captopril 25 mg every 24 hrs.
QUESTION No. 431
Considering the epidemiology of this disease is less
  frequent which of the following statements?

ANSWER
a. - may occur in up to 10% of pregnancies
b. - Some women may have signs of preeclampsia
   from week 20.
c. - Up to 85% of cases occur in primigravidae.
d. - The family history of hypertension.
QUESTION NO. 432
Which of conceptional risk factors for preeclampsia is
 more often seen?

ANSWER
a. - Recurrent urinary tract infection.
b. - Mean arterial pressure equal to or greater than 95
   mm Hg. in the second quarter.
c. - Excessive weight gain than expected for gestational
   age.
d. - Gestational Diabetes.
QUESTION No. 433
Which of the following factors is to develop HELLP
 poor prognosis for the patient?

ANSWER
a. - MT 160/110 mmHg.
b. - proteinuria greater than 2 g in 24 hrs urine.
c. - Creatinine> 1.2 and platelets <100,000.
d. - Increased liver enzymes
QUESTION No. 434
Which of the following preventive measures have the
 least evidence to reduce risk?

ANSWER
a.- Measure blood pressure after the 20th week.
b.- Control prenatal nutrition surveillance.
c.- Conduct studies of uric acid, creatinine and urea.
d.- EGO to search proteunuria (qualitative).
QUESTION No. 435
Which of the following preventive measures have the least
  evidence to reduce risk?

ANSWER
a.- Avoid stress: a pregnant Although not limited in your daily
   life if you must have relative rest.
b.- Calcium diet: Eat two or three grams of calcium from the
   first trimester of pregnancy appears to reduce the RISG of
   developing preeclampsia.
c.- Folic acid supplements: This substance is able to reduce the
   risk of preeclampsia by decreasing the concentration of
   homocysteine ​in the blood also prevents embryonic
   malformations such as spina bifida.
d.- Diet rich in antioxidants: Vitamin C and E are powerful
   antioxidants that eliminate free radicals and others that are
   part of the toxins.
CASE REPORT
Patient is 19 years old which comes first consultation
  for secondary amenorrhea for 30 weeks LMP, goes so
  far as unaware of their pregnancy at home, the
  patient complained of dizziness, abdominal pain
  stick, tightness of chest , headache and
  irritability, physical examination shows hyperemic
  sclera, mild facial redness, rot's increased and lower
  limb edema, proteinuria, fundus measured with 28
  cm, with decreased fetal activity, the vital signs of the
  mother were FR 21, FC 96, TA 130/100 3 times.
QUESTION No 436
What is the immediate behavior to continue the
 patient.

ANSWER
a. - Identify fetal viability.
b. - Search for HELLP syndrome.
c. - Magnesium sulphate 4 g IV.
d. - Prepare the patient for Caesarea.
QUESTION No. 437
Which of the following risk factors occurs most often?

ANSWER
a.- Primiparous or multiparous elderly and Age: <18 or>
   35 years
b.- Weight: <50 kg or obese (However you should
   consider that many obese women may have
   hypothyroidism, polycystic ovarian syndrome, High
   blood insulin levels and excess estrogen, leading causes
   of infertility or early abortion).
c.- Existence of chronic diseases: diabetes
   mellitus, hypertension, kidney disease, eye
   disease, collagen vascular disease (lupus
   erythematosus).
d.- Preeclampsa in a previous pregnancy
QUESTION No. 438
Which of the following is less common pathologies as
 diagnostic difference?

ANSWER
a.- Epilepsy.
b.- TBI.
c.- Subarachnoid hemorrhage.
d.- Ruptured cerebral aneurysm.
QUESTION No. 439
In which of the following conditions can be used
  magnesium sulphate?

ANSWER
a.- Torsades de pointes.
b.- WPW
c.- Priztmetal
d.- Brugadra.
QUESTION No. 440
Which of the following statements is less common in
 overdosage of magnesium sulfate.

ANSWER
a.- Dilated pupils, dry mouth, tachycardia, urinary
    retention, blurred vision.
b.- Behavioral
    changes, delirium, sedation, hallucinations, conv
    ulsions.
c.- Respiratory depression and coma.
d.- Decreased bowel movements and dehydration.
   Hyperpyrexia by inhibition of sweating.
Primer simulador segunda parte curso enarm cmn siglo xxi final
Primer simulador segunda parte curso enarm cmn siglo xxi final
Primer simulador segunda parte curso enarm cmn siglo xxi final
Primer simulador segunda parte curso enarm cmn siglo xxi final
Primer simulador segunda parte curso enarm cmn siglo xxi final
Primer simulador segunda parte curso enarm cmn siglo xxi final
Primer simulador segunda parte curso enarm cmn siglo xxi final
Primer simulador segunda parte curso enarm cmn siglo xxi final
Primer simulador segunda parte curso enarm cmn siglo xxi final
Primer simulador segunda parte curso enarm cmn siglo xxi final
Primer simulador segunda parte curso enarm cmn siglo xxi final
Primer simulador segunda parte curso enarm cmn siglo xxi final
Primer simulador segunda parte curso enarm cmn siglo xxi final
Primer simulador segunda parte curso enarm cmn siglo xxi final
Primer simulador segunda parte curso enarm cmn siglo xxi final
Primer simulador segunda parte curso enarm cmn siglo xxi final
Primer simulador segunda parte curso enarm cmn siglo xxi final
Primer simulador segunda parte curso enarm cmn siglo xxi final

Weitere ähnliche Inhalte

Was ist angesagt? (20)

Ulcera gastrica,duodenal completa
Ulcera gastrica,duodenal completaUlcera gastrica,duodenal completa
Ulcera gastrica,duodenal completa
 
Ulcus péptico y Qx
Ulcus péptico y QxUlcus péptico y Qx
Ulcus péptico y Qx
 
Preguntas ENARM cirugía clase 4
Preguntas ENARM cirugía clase 4Preguntas ENARM cirugía clase 4
Preguntas ENARM cirugía clase 4
 
Cancer De Colon
Cancer De ColonCancer De Colon
Cancer De Colon
 
CÁNCER DE ESÓFAGO
CÁNCER DE ESÓFAGOCÁNCER DE ESÓFAGO
CÁNCER DE ESÓFAGO
 
Invaginacion intestinal
Invaginacion intestinalInvaginacion intestinal
Invaginacion intestinal
 
Cuci
CuciCuci
Cuci
 
Cancer gastrico
Cancer gastricoCancer gastrico
Cancer gastrico
 
Incidentalomas
Incidentalomas Incidentalomas
Incidentalomas
 
Preguntas enarm c.c.l.
Preguntas enarm c.c.l.Preguntas enarm c.c.l.
Preguntas enarm c.c.l.
 
PERFORACIÓN DEL TRACTO GASTROINTESTINAL
PERFORACIÓN DEL TRACTO GASTROINTESTINALPERFORACIÓN DEL TRACTO GASTROINTESTINAL
PERFORACIÓN DEL TRACTO GASTROINTESTINAL
 
REVISION DE QUISTE COLEDOCO
REVISION DE QUISTE COLEDOCOREVISION DE QUISTE COLEDOCO
REVISION DE QUISTE COLEDOCO
 
Quiste de coledoco
Quiste de coledocoQuiste de coledoco
Quiste de coledoco
 
CUCI
 CUCI CUCI
CUCI
 
Examen de-rm-2013-parte-b
Examen de-rm-2013-parte-bExamen de-rm-2013-parte-b
Examen de-rm-2013-parte-b
 
Volvulo Intestinal
Volvulo IntestinalVolvulo Intestinal
Volvulo Intestinal
 
Tumores gastricos benignos
Tumores gastricos benignosTumores gastricos benignos
Tumores gastricos benignos
 
LITIASIS BILIAR
LITIASIS BILIARLITIASIS BILIAR
LITIASIS BILIAR
 
Esofagitis por ERGE
Esofagitis por ERGEEsofagitis por ERGE
Esofagitis por ERGE
 
Apendicitis enarm 2013
Apendicitis enarm 2013Apendicitis enarm 2013
Apendicitis enarm 2013
 

Andere mochten auch

Enarm 2145 preguntas
Enarm 2145 preguntasEnarm 2145 preguntas
Enarm 2145 preguntas3lsanto
 
Banco preguntas espejo 2014
Banco preguntas espejo 2014Banco preguntas espejo 2014
Banco preguntas espejo 2014Andy Guarnizo
 
Simulacro23. Las preguntas que no deberías fallar.
Simulacro23. Las preguntas que no deberías fallar.Simulacro23. Las preguntas que no deberías fallar.
Simulacro23. Las preguntas que no deberías fallar.Javier Camiña Muñiz
 
Crohn curso enarm cmn siglo xxi 36246001
Crohn curso enarm cmn siglo xxi  36246001Crohn curso enarm cmn siglo xxi  36246001
Crohn curso enarm cmn siglo xxi 36246001emilio2005angel1973
 
Hipertension arterial sistemica curso enarm cmn siglo xxi
Hipertension arterial sistemica curso enarm cmn siglo xxiHipertension arterial sistemica curso enarm cmn siglo xxi
Hipertension arterial sistemica curso enarm cmn siglo xxiemilio2005angel1973
 
Cam Xxv 2006 Cd3
Cam Xxv 2006 Cd3Cam Xxv 2006 Cd3
Cam Xxv 2006 Cd3enarm
 
ENFERMEDAD PELVICA INFLAMATORIA CURSO ENARM CMN SIGLO XXI 36246001
ENFERMEDAD PELVICA INFLAMATORIA CURSO ENARM CMN SIGLO XXI 36246001ENFERMEDAD PELVICA INFLAMATORIA CURSO ENARM CMN SIGLO XXI 36246001
ENFERMEDAD PELVICA INFLAMATORIA CURSO ENARM CMN SIGLO XXI 36246001Pharmed Solutions Institute
 
Preguntas de oncologia final
Preguntas de oncologia final Preguntas de oncologia final
Preguntas de oncologia final GRUPO D MEDICINA
 

Andere mochten auch (20)

Enarm 2145 preguntas
Enarm 2145 preguntasEnarm 2145 preguntas
Enarm 2145 preguntas
 
Primer simulador enarm 2013 primera parte final
Primer simulador enarm 2013 primera parte finalPrimer simulador enarm 2013 primera parte final
Primer simulador enarm 2013 primera parte final
 
enarm
enarmenarm
enarm
 
Simulador segundo ejemplo
Simulador segundo ejemploSimulador segundo ejemplo
Simulador segundo ejemplo
 
Banco preguntas espejo 2014
Banco preguntas espejo 2014Banco preguntas espejo 2014
Banco preguntas espejo 2014
 
Preguntas de tumor de ovario
Preguntas de tumor de ovarioPreguntas de tumor de ovario
Preguntas de tumor de ovario
 
SEGUNDO SIMULADOR (EJEMPLO)
SEGUNDO SIMULADOR (EJEMPLO)SEGUNDO SIMULADOR (EJEMPLO)
SEGUNDO SIMULADOR (EJEMPLO)
 
Varicela
VaricelaVaricela
Varicela
 
Simulacro23. Las preguntas que no deberías fallar.
Simulacro23. Las preguntas que no deberías fallar.Simulacro23. Las preguntas que no deberías fallar.
Simulacro23. Las preguntas que no deberías fallar.
 
Crohn curso enarm cmn siglo xxi 36246001
Crohn curso enarm cmn siglo xxi  36246001Crohn curso enarm cmn siglo xxi  36246001
Crohn curso enarm cmn siglo xxi 36246001
 
Hipertension arterial sistemica curso enarm cmn siglo xxi
Hipertension arterial sistemica curso enarm cmn siglo xxiHipertension arterial sistemica curso enarm cmn siglo xxi
Hipertension arterial sistemica curso enarm cmn siglo xxi
 
Crohn curso enarm cmn siglo xxi 36246001
Crohn curso enarm cmn siglo xxi  36246001Crohn curso enarm cmn siglo xxi  36246001
Crohn curso enarm cmn siglo xxi 36246001
 
Repaso final de oncologia
Repaso final de oncologiaRepaso final de oncologia
Repaso final de oncologia
 
Respuestas del segundo enarm version final
Respuestas del segundo enarm version finalRespuestas del segundo enarm version final
Respuestas del segundo enarm version final
 
Cam Xxv 2006 Cd3
Cam Xxv 2006 Cd3Cam Xxv 2006 Cd3
Cam Xxv 2006 Cd3
 
ENFERMEDAD PELVICA INFLAMATORIA CURSO ENARM CMN SIGLO XXI 36246001
ENFERMEDAD PELVICA INFLAMATORIA CURSO ENARM CMN SIGLO XXI 36246001ENFERMEDAD PELVICA INFLAMATORIA CURSO ENARM CMN SIGLO XXI 36246001
ENFERMEDAD PELVICA INFLAMATORIA CURSO ENARM CMN SIGLO XXI 36246001
 
Preguntas de oncologia final
Preguntas de oncologia final Preguntas de oncologia final
Preguntas de oncologia final
 
Crohn curso enarm cmn siglo xxi 36246001
Crohn curso enarm cmn siglo xxi  36246001Crohn curso enarm cmn siglo xxi  36246001
Crohn curso enarm cmn siglo xxi 36246001
 
Practica De Cariotipo
Practica De CariotipoPractica De Cariotipo
Practica De Cariotipo
 
Repaso enarm urgencias
Repaso enarm urgenciasRepaso enarm urgencias
Repaso enarm urgencias
 

Ähnlich wie Primer simulador segunda parte curso enarm cmn siglo xxi final

Parcial 1 Nehemías Jiménez 8-983-1010.docx
Parcial 1 Nehemías Jiménez 8-983-1010.docxParcial 1 Nehemías Jiménez 8-983-1010.docx
Parcial 1 Nehemías Jiménez 8-983-1010.docxturey20
 
Parcial 1 Nehemías Jiménez 8-983-1010.docx
Parcial 1 Nehemías Jiménez 8-983-1010.docxParcial 1 Nehemías Jiménez 8-983-1010.docx
Parcial 1 Nehemías Jiménez 8-983-1010.docxturey20
 
Parcial 1
Parcial 1 Parcial 1
Parcial 1 turey20
 
PREGUNTAS DEL EXAMEN NACIONAL DE MEDICINA 17/03/2024
PREGUNTAS DEL EXAMEN NACIONAL DE MEDICINA 17/03/2024PREGUNTAS DEL EXAMEN NACIONAL DE MEDICINA 17/03/2024
PREGUNTAS DEL EXAMEN NACIONAL DE MEDICINA 17/03/2024recas85266
 
Cuaderno 2003 1_m
Cuaderno 2003 1_mCuaderno 2003 1_m
Cuaderno 2003 1_mKitty578
 
CASO 1 Y 2 - SEMANA 15, LABORATORIO CLINICO
CASO 1  Y 2 - SEMANA 15, LABORATORIO CLINICOCASO 1  Y 2 - SEMANA 15, LABORATORIO CLINICO
CASO 1 Y 2 - SEMANA 15, LABORATORIO CLINICOPamela194327
 
Modulo 2 elaborando por Pedro Fernandez G. La Faye
Modulo 2  elaborando por Pedro Fernandez G. La FayeModulo 2  elaborando por Pedro Fernandez G. La Faye
Modulo 2 elaborando por Pedro Fernandez G. La FayeCurso Evaluación
 
CASOS CLINICOS APLICADOS EN LA PRACTICA.pptx
CASOS CLINICOS APLICADOS EN LA PRACTICA.pptxCASOS CLINICOS APLICADOS EN LA PRACTICA.pptx
CASOS CLINICOS APLICADOS EN LA PRACTICA.pptxShirleyPelez1
 
Colelitiasis hospital yopal
Colelitiasis hospital yopalColelitiasis hospital yopal
Colelitiasis hospital yopalMauricio Cortes
 
Caso clinico sindrome hipereosinofilico
Caso clinico sindrome hipereosinofilicoCaso clinico sindrome hipereosinofilico
Caso clinico sindrome hipereosinofilicoSheila Solano
 

Ähnlich wie Primer simulador segunda parte curso enarm cmn siglo xxi final (20)

Preguntas ENARM cirugia clase 2
Preguntas ENARM cirugia clase 2Preguntas ENARM cirugia clase 2
Preguntas ENARM cirugia clase 2
 
Parcial 1 Nehemías Jiménez 8-983-1010.docx
Parcial 1 Nehemías Jiménez 8-983-1010.docxParcial 1 Nehemías Jiménez 8-983-1010.docx
Parcial 1 Nehemías Jiménez 8-983-1010.docx
 
Parcial 1 Nehemías Jiménez 8-983-1010.docx
Parcial 1 Nehemías Jiménez 8-983-1010.docxParcial 1 Nehemías Jiménez 8-983-1010.docx
Parcial 1 Nehemías Jiménez 8-983-1010.docx
 
Parcial 1
Parcial 1 Parcial 1
Parcial 1
 
Disfagia
DisfagiaDisfagia
Disfagia
 
ESTENOSIS DE PILORO PRESENTACION
ESTENOSIS DE PILORO PRESENTACIONESTENOSIS DE PILORO PRESENTACION
ESTENOSIS DE PILORO PRESENTACION
 
patologia pancreo biliar.pdf
patologia pancreo biliar.pdfpatologia pancreo biliar.pdf
patologia pancreo biliar.pdf
 
PREGUNTAS DEL EXAMEN NACIONAL DE MEDICINA 17/03/2024
PREGUNTAS DEL EXAMEN NACIONAL DE MEDICINA 17/03/2024PREGUNTAS DEL EXAMEN NACIONAL DE MEDICINA 17/03/2024
PREGUNTAS DEL EXAMEN NACIONAL DE MEDICINA 17/03/2024
 
3
33
3
 
Cuaderno 2003 1_m
Cuaderno 2003 1_mCuaderno 2003 1_m
Cuaderno 2003 1_m
 
SEMANA 3 CIRUGÍA J5 (1) (1).pptx
SEMANA 3 CIRUGÍA J5 (1) (1).pptxSEMANA 3 CIRUGÍA J5 (1) (1).pptx
SEMANA 3 CIRUGÍA J5 (1) (1).pptx
 
(2018 10-23) ACALASIA.PPT
(2018 10-23) ACALASIA.PPT(2018 10-23) ACALASIA.PPT
(2018 10-23) ACALASIA.PPT
 
CASO 1 Y 2 - SEMANA 15, LABORATORIO CLINICO
CASO 1  Y 2 - SEMANA 15, LABORATORIO CLINICOCASO 1  Y 2 - SEMANA 15, LABORATORIO CLINICO
CASO 1 Y 2 - SEMANA 15, LABORATORIO CLINICO
 
Modulo 2 elaborando por Pedro Fernandez G. La Faye
Modulo 2  elaborando por Pedro Fernandez G. La FayeModulo 2  elaborando por Pedro Fernandez G. La Faye
Modulo 2 elaborando por Pedro Fernandez G. La Faye
 
CASOS CLINICOS APLICADOS EN LA PRACTICA.pptx
CASOS CLINICOS APLICADOS EN LA PRACTICA.pptxCASOS CLINICOS APLICADOS EN LA PRACTICA.pptx
CASOS CLINICOS APLICADOS EN LA PRACTICA.pptx
 
Colelitiasis hospital yopal
Colelitiasis hospital yopalColelitiasis hospital yopal
Colelitiasis hospital yopal
 
Neurofobromatosis
NeurofobromatosisNeurofobromatosis
Neurofobromatosis
 
Caso clinico sindrome hipereosinofilico
Caso clinico sindrome hipereosinofilicoCaso clinico sindrome hipereosinofilico
Caso clinico sindrome hipereosinofilico
 
COLECISTITIS AGUDA
COLECISTITIS AGUDACOLECISTITIS AGUDA
COLECISTITIS AGUDA
 
Cto.simulacro 8
Cto.simulacro 8Cto.simulacro 8
Cto.simulacro 8
 

Mehr von Pharmed Solutions Institute (20)

Sarampion
SarampionSarampion
Sarampion
 
Rubeola
RubeolaRubeola
Rubeola
 
Quemadurasbuena
QuemadurasbuenaQuemadurasbuena
Quemadurasbuena
 
Parotidit
ParotiditParotidit
Parotidit
 
Meningitisss
MeningitisssMeningitisss
Meningitisss
 
Leucemias
LeucemiasLeucemias
Leucemias
 
Influenza
InfluenzaInfluenza
Influenza
 
Infecciones respi
Infecciones respiInfecciones respi
Infecciones respi
 
Neumonías
NeumoníasNeumonías
Neumonías
 
Hepatitis
HepatitisHepatitis
Hepatitis
 
Escarlatina
EscarlatinaEscarlatina
Escarlatina
 
Vivora
VivoraVivora
Vivora
 
Araña
ArañaAraña
Araña
 
Alacraan
AlacraanAlacraan
Alacraan
 
Parasitos
ParasitosParasitos
Parasitos
 
Diarrea
DiarreaDiarrea
Diarrea
 
Vih sida
Vih sidaVih sida
Vih sida
 
Tuberculosis
TuberculosisTuberculosis
Tuberculosis
 
Síndrome de sjogren
Síndrome de sjogrenSíndrome de sjogren
Síndrome de sjogren
 
Moraxella catarrhalis
Moraxella catarrhalisMoraxella catarrhalis
Moraxella catarrhalis
 

Primer simulador segunda parte curso enarm cmn siglo xxi final

  • 1. PRIMER SIMULADOR CURSO ENARM CMN SIGLO XXI INICIO DE LA SEGUNDA PARTE
  • 2. CASO CLINICO Se trata de paciente de 49 años con perdida de peso de mas de 15 % en 4 meses, acompañado de ictericia y dolor en la región epigástrica, tiene el antecedente de alcoholismo de 30 años de duración, tabaquismo positivo, hace 2 años fue tratado en varias ocaciones por gastritis, a la EF se palpa vesícula no dolorosa, indurada, se observa con tromboflebitis.
  • 3. PREGUNTA No. 251 Considerando el diagnostico presuntivo que ha considerado, cual es signo mas frecuente observar? RESPUESTA a.- Ictericia. b.- Perdida de peso. c.- Vesícula no dolorosa. d.- Esteatorrea
  • 4. PREGUNTA No. 252 Considerando el diagnostico presuntivo que ha considerado, cual es síntoma mas frecuente observar? RESPUESTA a.- Distención abdominal. b.- Dolor abdominal epigástrico sordo. c.- Dolor abdominal epigastrico difuso. d.- Prurito generalizado.
  • 5. PREGUNTA No. 253 Considerando el diagnostico presuntivo cual es el estudio de elección para apoyar el diagnostico? RESPUESTA a.- IRMf. b.- TAC helicoidal. c.- USG. d.- Gamagrama.
  • 6. PREGUNTA No. 254 Considerando el diagnostico considerado por usted, cuales es la metástasis primaria mas frecuente? RESPUESTA a.- Hepático. b.- Ganglios linfaticos. c.- Peritoneo. d.- Pulmón
  • 7. PREGUNTA No. 255 Considerando las caracteristicas e incidencia cual es la neoplasia mas probable? RESPUESTA a.- Cistoadenoma mucinoso b.- Adenocarcinoma pancreatico c.- Cistoadenomas ceroso. d.- Neoplasia quistica solido papilar.
  • 8. CASO CLINICO Se trata de paciente femenino de 65 años de edad que acude a consulta por dolor difuso, perdida de peso, refiere que ha presentado cuadros de mareo, piel fría, sudoración, frio, con desorientación, durante estos cuadros ha disminuido los síntomas con el consumo de dulces, a la EF se destaca taquicardia, diaforesis y glucosa periférica de 50.
  • 9. PREGUNTA No. 256 El paciente presenta la triada de wipple, cual es el método diagnostico inicial para considerar su localización? RESPUESTA a.- USG. b.- TAC. c.- IRM. d.- Doppler transoperatorio.
  • 10. PREGUNTA No. 257 Cual de los siguientes padecimientos no es diagnostico diferencial? RESPUESTA a.- VIPoma. b.- Gastrinoma c.- Insulinoma. d.- Papiloma
  • 11. CASO CLINICO Se trata de paciente masculino, que acude por cansancio, fatiga, así como dolor en hipocondrio derecho, anemia aguda, se realizo USG de hígado y vías biliares donde se apreciaron cálculos biliares, no se aprecia ictericia sin embargo, se realizo estudios de laboratorio observando glucosa de 190, el paciente refiere que no era diabético, Anemia normocromica megaloblastica, durante su estudio el paciente perdió mas de 20 % de su peso y su estado se deterioro severamente, su piel se observa aspera, esteatorrea e hipocloridria.
  • 12. PREGUNTA No. 258 Cual es la conducta mas apropiada a seguir? RESPUESTA a.- Dieta hiperproteica y complejo B. b.- Hierro suplementario y complejo B c.- Suplementacion enzimatica. d.- Tomografia axial computada.
  • 13. PREGUNTA No. 259 Considerando los datos clínicos, usted solicito valores de somatostatina encontrándose en 130 mcg/dl, se diagnostico somatostinoma, cual es la conducta? RESPUESTA a.- Quimioterapia y posterior resección. b.- Embolización y radioterapia. c.- Realizar Wipple. d.- Braquiterapia previa em
  • 14. CASO CLINICO Se trata de paciente de 7 años con dilución, disfunción y atrofia, edema caída de cabello, anorexia y diarrea, lesiones queratosis, seca, ictiositosica, además lesiones hipercromica, lesiones purpuricas, asi como petequias, piel marmorea con telangiectacias.
  • 15. PREGUNTA No. 260 Cual es su impresión diagnostica inicial? RESPUESTA a.- Purpura. b.- Leucemia. c.- Desnutrición. d.- Fibrosis quistica
  • 16. PREGUNTA No. 261 Considerando el protocolo de estudio para apoyar su diagnostico cuales son los estudios de laboratorio y gabinete mas adecuado? RESPUESTA a.- BH, EGO, ES, PFH, COPRO. b.- EGO, ES, BH, PFR, PFH. c.- BH, Frotis de sangre, EGO, Cultivos, PFH. d.- Perfil lipidos, Frotis de sangre, EGO, BH, PFH.
  • 17. PREGUNTA No. 262 Cual es la manifestación inicial de laboratorio que se presenta en el paciente con desnutrición? RESPUESTA a.- Anemia. b.- Hipoalbuminemia. c.- Leucopenia. d.- Plaquetopenia.
  • 18. PREGUNTA No. 263 El paciente presenta en una segunda consulta las siguientes manifestaciones, diarrea, anorexia palidez, sin ganas de jugar, que tipo de manifestaciones son? RESPUESTA a.- Dilucionales b.- Circuntanciales c.- Agregados. d.- Comorbidos
  • 19. CASO CLINICO Paciente de 15 años Presentaba hipogonadismo hipogonadotrófico, retardo mental y IMC=57,7 kg/m2. El paciente presentaba serios problemas de higiene, micosis en pliegues y múltiples problemas sociales. Había intentado a lo largo de su vida variados tratamientos médicos para reducción de peso (dietas, programa de ejercicio físico, uso de anfetaminas, etc.) sin éxito permanente, ya que debido al componente hiperfágico de su síndrome de base no lograba mantener la disminución ponderal.
  • 20. PREGUNTA No 264 Considerando las manifestaciones clínicas, se realizo un cariotipo 46 XY del (15) (q11q13)? RESPUESTA a.- Prader – willi. b.- Sindrome de carpinter. c.- Sindrome de cohen. d.- Sindrome Down.
  • 21. PREGUNTA No 265 Considerando el cuadro clínico, cual es la conducta a seguir? RESPUESTA a.- Dieta b.- Ejercicio c.- Estilo de vida. d.- Y de Roux
  • 22. PREGUNTA No. 266 Cual es el tratamiento farmacológico a largo plazo con evidencia y aprobación para el paciente considerando su edad? RESPUESTA a.- Fentermina b.- Metfomina c.- Sibutramina d.- Orlistat
  • 23. CASO CLINICO Una mujer de 45 años presenta dolor en hipocondrio derecho de 36 hrs. de evolución. TGP: 80, amilasemia: 250 ui. La ecografía abdominal informa: quiste de 3 cm de diámetro en segmento III del hígado, pared vesicular de 5 mm trilaminar, con un diámetro vesicular transverso de 106 mm y un cálculo impactado en cuello.
  • 24. PREGUNTA No. 267 Cual es la conducta a seguir mas adecuada? RESPUESTA a.- Hidratación, analgésicos, antibiótico y derivación por consulta externa para cirugía programada b.- Hidratación, analgésicos, antibiótico y cirugía abierta de urgencia. c.- Hidratación, analgésicos, antibiótico y laparoscopía a partir de las 72 hs de internamiento. d.- Hidratación, analgésicos, antibiótico y laparoscopía antes de las 72 hs de internamiento.
  • 25. CASO CLINICO Una paciente de 74 años, apendicectomizada a los 10 años y sin antecedentes médicos de importancia ingresa a la guardia por vómitos, distensión abdominal y falta de eliminación de gases de 48 horas de evolución. En la radiografía de abdomen (ver imagen), el cuadro del paciente se ha presentado recurrentemente en los 5 últimos años.
  • 26.
  • 27. RESPUESTA No. 268 ¿Cuál es la patología obstructiva más probable en este cuadro? RESPUESTA a.- Cáncer de colon derecho b.- Bridas de intestino delgado c.- Fecaloma d.- Constipación crónica pertinaz
  • 28. CASO CLINICO Se trata de paciente masculino de femenino de 36 años de edad, con sobrepeso que reingreso posterior a un mes de tratamiento por quemaduras de 3 grado, acude con dolor en hipocondrio derecho, el cual se activa con la alimentación, predominantemente con grasas, el ultrasonido se muestra a continuación. A la exploración física se encuentra estable signos vitales dentro de rango de normalidad, solo destaca murphy positivo, con puntos uretrales altos igualmente positivos, PFH sin datos patológicos.
  • 29.
  • 30. PREGUNTA No. 269 Cual es la conducta a seguir? RESPUESTA a.- Realizar cirugía laparoscópica. b.- Hidratación, analgesia, antibiótico. c.- Realizar CPRE. d.- Alta y envío a consulta externa de cirugía.
  • 31. PREGUNTA No. 270 Cuales son los agentes infecciosos más frecuentes que se pueden encontrar en el presente caso? RESPUESTA a.- Eschericia coli. b.- Klebsiella penumoniae. c.- Steptococcus faecalis. d.- Bacteroides fragilis
  • 32. PREGUNTA No. 271 El paciente inicia con fiebre súbita de 39.7 grados, con ataque al estado generalizado, dolor en hipocondrio derecho que se irradia a en barra y es de características trasfictivo, se observo un incremento de DHL, PCR y VSG así como leucocitosis de 21,000 e hiperglucemia de 250 mg/dl, cual es la conducta mas adecuada a seguir? RESPUESTA a.- Realizar LAPE. b.- Hidratación, morfina y antibiótico. c.- Realizar CPRE. d.- Realizar TAC.
  • 33. CLINIC CASE An 80 year old man has a 20 month history of progressive gait disturbance and incontinence, which had been attributed to old age and prostatism. His gait is dhort-stepped, and he turns very slowy, almost toppling over.
  • 34. QUESTION No. 272 The most likely diagnosis is? ANSWER a.- Chiari malformation. b.- Cruetzfeldt-Jacob disease. c.- Progressive multifocal leukoencephalopathy. d.- Normal pressure hydrocephalus.
  • 35. CLINIC CASE A 29 year old man presented with sudden onset low back pain with radation to his right leg while attempting to lift a box. The neurological exam showed normal muscle strength in his legs bilaterally with preserved anal tone.
  • 36. QUESTION No. 273 The best management at this point is? ANSWER a.- Acumpunture referral. b.- Bed rest with NSAID´s. c.- CT scan of the lumbar spine. d.- Stat MRI of the lumbar spine.
  • 37. CLINIC CASE A 52 year old female presented to the emergency room with sudden onset of mental status change associated with left sided weakness. Initial CT head without contrast showed right temporal intracerebral hemorrhage. CT angiogram of the head is shown below.
  • 38.
  • 39. QUESTION No. 274 Based on the imaging findings, what is the likely etiology for this intracerebral hemorrhage? ANSWER a.- Hemorrhagic conversion of ischemic stroke. b.- Posterior comunicanting artery aneurysm. c.- Hypertensive cerebral hemorrhage. d.- Dural sinus thrombosis.
  • 40. CLINICAL CASE A 70 year old male was brougth to the emergency room by his wife to be evaluated for his agitation. The patient went to bed last night with normal behavior. He awoke this morning confussed with visual hallucinations. The patient was found to have an ischemic stroke associated with agitated delirium.
  • 41. QUESTION No. 275 Which of the following arteries is responsible for his stroke? ANSWER a.- Left anterior inferior cerebral artery. b.- Left middle cerebral artery. c.- Left posterior cerebral artery. d.- Left posterior inferior cerebellar artery.
  • 42. CLINIC CASE A 70 year old patient with past medical history of hypertension and diabetes, complains of sudden onset visual disturbance. Neurological examination reveals right homonymous hemianopia with macula sparing.
  • 43. QUESTION No. 276 What is the localization of this ischemic stroke? ANSWER a.- Left occipital lobe. b.- Right occipital lobe. c.- Left parietal lobe. d.- Right parietal lobe.
  • 44. CLINIC CASE A 55 year old man had a transient episode of left sided weakness that lasted 45 minutes. The patient was evaluated at the emergency department with a CT scan of the head w/o contrast, which was normal.
  • 45. QUESTION No. 277 What factors should be considered regarding the decision for impatient vs outpatient workup? ANSWER a.- AII TIA patients should be worked up as outpatient. b.- ABCD2 score should be considered. c.- The patient vs outpatient workup is the neurologist´s preference. d.- No need for extensive workup as the symptoms resolved completely.
  • 46. CLINIC CASE An 83 year old woman with hypertension and atrial fibrillation on coumadin presented with an acute onset of left-sided weakness. Her daughter called an ambulance. The patient was evaluated in the emergency room and CT head was obtained (see image).
  • 47.
  • 48. QUESTION No. 278 What is the diagnosis? ANSWER a.- Subdurachnoid hemorrhage. b.- Intracranial hemorrhage. c.- Subdural hematoma. d.- Epidural hematoma.
  • 49. CLINIC CASE A 4 year old male presents to your office for evaluations of his high fever. He was seen 2 days ago at your office with ear pain and a temperature of 100.5 F. At that time, noth tympanic membranes were bulging, erythematous, and opaque. He was prescribed ab oral antibiotic for bilateral otitis media. The mother states that she was unable to get the prescription filled until this morning, Vital signs in the ofice include an oral temperature of 39.4 C. Both heart rate and respiratory rate are increased, but not over what would be expected based on the degree of fever. Otoscopy finding are unchanged. The left pinna is noted to be protruding laterally.
  • 50. QUESTION No. 279 This patient is most likely to experience tenderness to palpation in which of the following anatomic areas? ANSWER a.- Left mastoid region. b.- Left paranasal sinuses. c.- Left temporal region. d.- Left lower molars
  • 51. CLINIC CASE A 3 year old females is brought to your office for evaluation of fever and a rash. She was in her usual state of goog health until the day before yesterday, when she developed a fever to 38.6 C. For the past two days she has rested more than usual and been fussy but consolable.
  • 52. Last night the mother noticed red bumps on her face. This morning those lesions looked like fluid-filled blisters, and she has new red bumps on her chest and upper arms. The mother tells you that she has tried to kepp her daughter from scratching them because she doesn´t want them to spread. She has scratched one of the lisions on her face so much that if has broken open and crusted over.
  • 53. QUESTION No. 280 Which pf the following represents the most appropriate management of the patient at this time? ANSWER a.- Caution the mother not to become pregnant during the childs illnes. b.- Assure the parent that the rash will resolve on its own without specific tratment. c.- Prescribe asperin to control the fever. d.- Prescribe oral ketoconazole and penicillin.
  • 54. CLINIC CASE A 3 year old female presents with a fever of 102.7 F, chills, and vomiting. She had been in her usual state of good health until yesterday, when she became ill with what her parents thought was a gastrointestinal illnes. Review of systems is negative for rhinorrhea, cough, ear pain, sore throat, diarrhea, and rash. Her past medical history is significant for 2 prior episodes of cystitis, both of which resolved with oral antibiotics.
  • 55. Micrioscopic examination of the urine reveals numerous white blood cells, red blood cells and gram negative rods. Elevation of the erythrocyte sedimentation rate suggests involvement of the upper urinary tract. The patient is admitedd to the hospital for intravenous and fluids.
  • 56. QUESTION No. 281 Which of the following is the most likely cause of this patient´s recurrent urinary tract infections? ANSWER a.- Polycystic kidney disease. b.- Posterior urethral valves. c.- Vesicoureteral reflux. d.- Primary b-cell inmunodeficiency.
  • 57. CLINIC CASE An 8 year old boy was at an end of the year school picnic when he was stung by a flying insect. Within minutes of being stung, the youngster began to have difficulty breathing. Paramedics were called to the scene. The frightened child was found to be moderately hypotensive, wheezing, and in significant respiratory distress. Epinephrine was administered at the scene, and the patient was placed on axygen via face mask.
  • 58. Upon arrival to the emergency department, the patient received a dose each of an intravenous antihistamine and intravenous corticosteroid. He was observed overnigth in the hospital and discharged the next day after the parents had obtained injectable epinephrine and been taught to administer it. He is in your office today for futher education and follow-up care. He has a positive skin prick test for yellow jacket venom.
  • 59. QUESTION No. 282 Which of the following represents the most appropriate next step in the management of the patient? ANSWER a.- Daily administration of a non-sedating antihistamine throughout the spring, summer an fall months. b.- Skin prick testing for hornet venom sensitivity. c.- Allergen immunotherapy. d.- Specific serum honeybee IgE antibody testing.
  • 60. CLINIC CASE A 5 year old female is in your office for her yearly health maintenance visit, She is in her usual state of good health. Height and weight continue to track along pre-estabilished curves, both at the 50th percentiles, her mother describes her as active and curious. While examining her chest, you note a mid-systolic grade II/VI vibratory murmur heard best at the left lower sternal border.
  • 61. The murmur intensifies in the supine position. There are no associated heaves, thrills, clicks or rubs. The second heart is normally split with inspiration and becomes singles on full expiration. Femoral pulses are symmetric and not delayed. No cyanosis or clubbing is noted on examination of the extremities.
  • 62. QUESTION No. 283 Which of the following is the most likely diagnosis? ANSWER a.- Still´s murmur. b.- Atrial septal defect. c.- Patent ductus arteriosus. d.- Tetralogy of fallot.
  • 63. CLINIC CASE A 3 week old former full-term infant is in your office for her regular health maintenance visit. She is growing and developing of the right leg with asymmetric gluteal folds. When gentle pressure is applied to the flexed and adducted rigth hip in a posterior direction, there is no detectable clunk: however, when this is followed by hip abduction, a palpable clunk ensure.
  • 64. Preinatal history is significant for cesarean delivery secondary to persistent brech presentation. The is a result of the mother´s first pregnancy.
  • 65. QUESTION No. 284 Which of the following is the most likely explanation for these findings? ANSWER a.- Metatarsus adductus. b.- Tlipes equinovarus. c.- Developmental dysplasia of the hip. d.- Osgood-Schlatter disease.
  • 66. CLINIC CASE A 33 year old woman who is obese (BMI 37) comes to your office because she heard in the news that obesity increases the risk for cancer. QUESTION No. 285 You advise that patient that obesity does indeed increase the risk for all of the following cancer in woman of her age except? ANSWER a.- Breast cancer. b.- Endometrial cancer. c.- Gallbladder cancer. d.- Renal cell cancinoma.
  • 67. CLINIC CASE In counseling a 39 year old obese BMI 34, man. QUESTION No 286 You inform him he is at increased risk of all of the following except? ANSWER a.- Galistones. b.- Slepp apnea. c.- Osteoarthritis. d.- Glaucoma.
  • 68. CLINIC CASE A 24 year old woman with history of Hashimoto´s thyroiditis come to a follow up visit and informs you that she is pregnant. Her gynecologist has started her on prenatal vitamins but advised her to see you because of her thyroid disease.
  • 69. QUESTION No. 287 Which of the following is true for this patient? ANSWER a.- She has a low risk for developing pre-eclampsia during her pregnancy. b.- Her thyroid antibodies are likely to increase during the pregnancy. c.- She will need to progressively increase her dose of levothyroxine during her pregnancy. d.- Her likelihood of undergoing a cesarean section is decreased.
  • 70. CLINIC CASE A 31 year old man with history of type 1 diabetes sice the age of 4 present to your office for routine follow up, the patient´s A1c is 7.2% and he is using an insulin aspart for diabetes control. On physical exam, you find that the patient has a blood pressure of 142/86 mmHg. This is the first time his bloog pressure readings have been elevated. A spot urine albumin/creatinine done one month prior is 39.
  • 71. You have the patient return a week later to recheck blood pressure and review new laboratories. Now his blood pressure is 149¿6/84 mmHg and he has a spot urine albumin/creatinine of 36. his creatinine is 1.3 mg/dL. It had been 1.1 mg/dL. 6 momths prior. You start the patient on enalapril and have him return two weeks later with blood chemistries. At the next follow up visit, the patient´s blood pressure is now 152/88 mmHg and his creatinine is 1.7 mg/dL, and the potassium is 5.1 mEq/L.
  • 72. QUESTION No. 288 Which of the following shold be the nest step in the treatment of his patient? ANSWER a.- Order a gadolinium-enhanced MRA. b.- Order a renal duplex ultrasound. c.- Discontinue enalapril and start verapamil. d.- Increse his dose of enalapril
  • 73. QUESTION No. 289 Which of the following is necessary to prepare a patient who will be undergoing resection of a medulary adrenal mass which has been confirmed to be a pheochromocytoma? ANSWER a.- Liberalize salt intake(high sodium diet) at least 10 days prior to surgery. b.- Discontinue antihypertensive therapy 3 days prior to surgery. c.- Liberalize sal intake (high sodium diet) at least 3 days prior to surgery. d.- Discontinue antihypertensive therapy 1 day prior to surgery.
  • 74. CLINIC CASE A 58 yer ol man presents to your office as a new patient. H has nt seen a physician in several years. The patient states that he is getting up at night frequently to urinate. He is thirsty most of the time, and he has blurry vision. He has also been bruising easily and has notice he looks mor tanned.
  • 75. On physical examn his pulse is 82 bpm and his blood pressure is 166/92 mmHg. The adbomen is large with prple striae and extremities are thin with several ecchymoses. You ask the patient to squat and stand up but he is unable to stand up without assistance. The patient´s skin shows darker pigmentation in skin fold areas. You order laboratories and radiological diagnostic and confirm a diagnosis of cushung´s disease.
  • 76. QUESTION No. 290 Which of the following is recommended for this patient? ANSWER a.- Adrenalectomy. b.- Gamma knife tratment. c.- Ketoconazole therapy. d.- Transphenoydal microsurgery.
  • 77. QUESTION No. 291 All of the following patient have an increassed risk for secondary osteoporosis except? ANSWER a.- A 47 year old woman with history of asthma who uses inhaled steroids. b.- A 38 year old women with history of epilepsy who requires medication to continue to suppress seizure activity. c.- A 42 year old man with bipolar disorder who is on chronic lithium therapy. d.- A 52 year old men with history of hypothyroidism who has a THS of 1.1 mIU/L.
  • 78. CLINIC CASE A 64 year old man has been diagnosed with papillary thyroid carcinoma. The patient underwent total thyroidectomy and recerved 100 mci of 1131 for ablation of any remaining tissue. A thyroglobulin level done one week after ablation is less than 1 ng/mL. You need to start thyroid hormone as adjunvant therapy for the thyroid cancer.
  • 79. QUESTION No. 292 What should your goal for this patient´s TSH be? ANSWER a.- < 0.1 uIU/mL. b.- < 0.01 uIU/mL. c.- < 0.4 uIU/mL. d.- < 1.0 uIU/mL.
  • 80. CASO CLINICO Se trata de paciente femenino de 26 años de edad que refiere dolor desde hace 3 años el cual se ha incrementado paulatinamente, el dolor se presenta principalmente durante la menstruación, acudió a consulta por que ahora es incapacitante disminuye levemente con el uso de AINE´s y antiespasmoliticos, niega leucorrea, no hay datos de irritación peritoneal sin embargo si presenta defensa. Casada desde hace 4 años y sin método anticonceptivo.
  • 81. PREGUNTA No. 293 Considerando su impresión diagnostica, cual es su conducta mas adecuada? RESPUESTA a.- Cefotaxima. b.- Antifungicos. c.- Indometacina. d.- Progestagenos
  • 82. PREGUNTA No. 294 La paciente regresa 3 meses después refiriendo disminución significativa del dolor, con disminución de la cantidad de menstruación comparada con las anteriores, considerando la respuesta del paciente, cual es el diagnostico mas probable del caso? RESPUESTA a.- Endometriosis. b.- Tuberculosis tubarica. c.- Enfermedad pélvica inflamatoria. d.- Síndrome de colon irritable.
  • 83. PREGUNTA No. 295 La paciente acude a revisión refiriendo mejoría significativa sin embargo usted observa un incremento en su peso, sus pligues se encuentran obscurecidos, no muestra estrias violaceas ni edema de miembros inferiores, pero si hay acne e hirsutismo leve, cual es la conducta a seguir? RESPUESTA a.- Metformida. b.- Analogos GnH. c.- Antiandrogenos. d.- Glipicida
  • 84. CASO CLINICO Se trata de femenino de 14 años de edad con tanner de acuerdo a edad, no cuenta con antecedentes de importancia, sin vida sexual activa, acudió por flujo vaginal anormal, por el interrogatorio hay datos que corresponden a episodios de opsomenorrea con retrasos de 30 a 50 dias, sin que ocurran síntomas asociados.
  • 85. PREGUNTA No. 296 Considerando el cuadro clínico, cual es su conducta a seguir? RESPUESTA a.- Uso de supresión de progesterona. b.- Uso de anticonceptivos orales combinados. c.- Se solicita perfil ginecologico. d.- Se solicita ultrasonograma.
  • 86. CLINIC CASE You are asked to evaluate a 32 year old men who presents with a sharp pain at the front of the neck which radiates to the jaw. The patient denies ane fever but does say that he is fatigued and has some trouble swallowing since the pain started. He has been recovering from an influenza infection over the past week. Upon examination of the neck, there is tenderness over they thyroid qland with gentle pressure.
  • 87. QUESTION No 297 Which of the following findings would you not expect to have for this patient? ANSWER a.- Thyroid uptake of 1%. b.- low thyroglobulin level. c.- fT4 level 6.4 ng/dL. d.- TSH level 0.8 mlU/L.
  • 88. CLINIC CASE A 24 yer old woman comes to your office to ask about her risk for diabetes. She delivered a baby 2 months ago and during her pregnancy she developed gestational diabetes. Her glucose levels are now after a 2 hour glucose tolerance test.
  • 89. QUESTION No. 298 You advice this patient that? a.- She doesn´t need to be concerned as GDM is reversible an does no increase the risk for diabetes. b.- She should begin a medical nutrition therapy program in addition to exercise since her risk for developing diabetes over the next 5 years is 50 % c.- She sould begin a medical nutrition therapy program in addition to excercise since her risk for developing diabetes over the next 5 years is 25%. d.- She should begin therapy with sitaglipin since she already has diabetes.
  • 90. CLINIC CASE You are evaluating a 74 year old woman who resides in a nursing home because of physical limitations due to a several strok she suffered a rear prior. The patient has been losing weight steadily over the past year due to depression and poor food intake. The nurse who follows the patient reports that the patient has been having a facial twitch. Upon examination, you are able to elicit trousseau´s sign. A calcium level done a week prior was 7.5 mg/dL.
  • 91. QUESTION No. 299 Which test should be ordered to verify the cause of the patient´s hypocalcemia? ANSWER a.- 1,25 (OH)2 vitamin D. b.- PTH. c.- 25 (OH) vitamin D. d.- Magnesium.
  • 92. CLINIC CASE A 61 year old woman with osteoporosis come to follow up after she has been on alendronate for 2 years. Her BMD when therapy was initiated was -3.8. her new BMD shows her BMD to be -3.6. The pantient currently takes 1000 Units of vitamin D and 2000 mg of calcium daily. Her vitamin D levels are normal and she has no histoty of frantures.
  • 93. QUESTION No. 300 Which of the following would you recommend to this patient? ANSWER a.- Discontinue alendronate and estart teriparatide. b.- Continue alendronate and increase vitamin D to 2000 Units/day. c.- Discontinue alendronate and start raloxifene. d.- Continue alendronate and add serum calcitonin.
  • 94. CLINIC CASE A 52 year old male from monterrey reports a history of lyme disease and subsequent polyartheritis. He was treated with antibiotics 3 months ago and has been asymptomatic. He went back to this primary care phymary care physician who retested him for lyme disease. He was found to have two IgM bands and five IgG bands (a positive test) on western blot.
  • 95. QUESTION No. 301 Which of the following should he do next? ANSWER a.- Ignore the results. b.- Recheck lab in 3 months. c.- Refer for synovectomy. d.- Doxycycline 100 mg bid for 28 days.
  • 96. CLINIC CASE A 37 year old male comes to the emergency department with ongoing fever upto 102 F for the past 4 weeks. He has an associated diffuse nonpruritic salmon- colored maculopapular skin rash. He also complains of enlarged lymph nodes, bilateral wrist swelling, and muscle aches. Laboratory testing a negative antinuclear antibody and rheumatoid factor?
  • 97. QUESTION No. 302 Which of the following is the most likely diagnosis? ANSWER a.- Becet´s diesease. b.- Systemic lupus erythematosus. c.- Palindromic rheumatism. d.- Adult stull´s disease.
  • 98. CLINIC CASE You are consulted on a 72 year old female in the hospital for arthitis. She has a long history of bilateral hand paind. Her examination reveals subluxation of her MCP´s, ulnar deviation, and swan neck and boutineers deformities. There is significant joint tenderness of her MCPs and PIPs and she has splenomegaly. Her labs show a WBC count of 1500 and hemoglobin of 10 g/dL with a normal MCV. The ESR is 74 and a CRP is 16.1 mg/dl.
  • 99. QUESTION No. 303 Which of the following is the most appropriate treatment at this time? ANSWER a.- Methotrexate. b.- Splenectomy. c.- Cyclophosphamide. d.- Neupogen.
  • 100. QUESTION No. 304 Which of the following is the less appropriate treatment at this time? ANSWER a.- Methotrexate. b.- Splenectomy. c.- Cyclophosphamide. d.- Neupogen.
  • 101. CASO CLINICO Una mujer de 74 años de edad se presenta con dolor tipo cólico y distensión abdominal. Su H.C es notable por obesidad. Ella nunca ha tenido un episodio previo de dolor abdominal como este, pero sufre de ataques de dolor en cuadrante superior derecho después de comidas colecistocinéticas. Ella está afebril, normotensa, icterica, FC de 95, con dolor a la palpación profunda en hipocondrio derecho, murphy y pronn. USG con VB de 10 cm, pared de 4 mm, Colédoco de 12 mm.
  • 102. PREGUNTA No. 305 Cual es el tratamiento mas adecuado al ingreso del paciente? a.- Colecistectomía laparoscópica b.- Realizar CPRE c.- Analgésico, antibiótico y rehidratación d.- Solicita PFH, QS, BH, tiempos de coagulación.
  • 103. CASO CLINICO Una mujer de 50 años se presenta con una historia de cuatro semanas de evolución, con dolor cólico en cuadrante superior derecho que se irradia a omoplato derecho, se encuentra en posición en gatillo, se observa levemente agitada, con fiebre de 37.9 C, su IMC 36, Colesterol 433, trigliceridos 524, bilirrubinas se encuentra normales, Se encuentra con mucosas deshidratadas, no hay datos de irritación peritoneal, niega antecedentes de dolor posterior a colecistoquinetico, se realizo un USG que no demuestra evidencia de colelitiasis o de coledocolitiasis.
  • 104. PREGUNTA No. 306 ¿Considerando los antecedentes cual es la conducta mas apropiada a seguir? RESPUESTA a.- Morfina, ceftriazona y solución mixta. b.- Preparar para realizar CPRE. c.- Laboratorios QS, EGO, BH, TPT y PFH. d.- Antiespasmodico, hidratación, antibiótico y laboratorios.
  • 105. CASO CLINICO Se trata de un paciente masculino de 38 años el cual acude al servicio de urgencias por opresión en tórax, no transitivo, taquicardia, dolor abdominal, refiere encontrarse bajo mucha presión laboral.
  • 106. PREGUNTA No. 307 Tomando los datos obtenido cual es su conducta a seguir? a.- Radiografia de tórax, ECG y enzimas. b.- Oxigeno, analgesico y laboratorios c.- Ansiolitico, sedante e hipnotico. d.- Oxigeno, analgesico y ansiolitico
  • 107. CASO CLINICO Se trata de masculino de 51 años de edad el cual acude por perdida de peso de 15 kilos en 3 meses, acompañado de dolor epigástrico continuo, el paciente tiene antecedentes de tabaquismo y alcoholismo crónico, así como gastritis y colitis de larga evolución, con múltiples tratamiento con moderado efecto. Sus laboratorios presentaron bilirrubina total de 21, plaquetopenia, anemia aguda y PFH elevadas.
  • 108. PREGUNTA No. 308 Considerando su diagnostico actual, cual es la tipo mas probable en este caso? RESPUESTA a.- Insulinoma. b.- Gastrinoma. c.- VIPoma. d.- Somastostinoma.
  • 109. PREGUNTA No. 309 Considerando su diagnostico, con la clasificación de este grupo de patología que tipo es? RESPUESTA a.- Primitivos. b.- Primarios. c.- Secundarios d.- Metastasicos.
  • 110. PREGUNTA No. 310 Considerando la pregunta anterior cual es el subtipo mas probable? RESPUESTA a.- Epiteliales sólidos. b.- Epiteliales mixtos. c.- Epiteliales quístico. d.- Epiteliales musinoso.
  • 111. PREGUNTA No. 311 Considerando el diagnostico establecido, cual de los siguientes localizaciones en la segunda mas frecuente? RESPUESTA a.- Colon b.- Pancreas c.- Gastrico. d.- Esofagico.
  • 112. PREGUNTA No. 312 Cual de los siguientes tumores es tiene tratamiento paliativo únicamente? RESPUESTA a.- Colon b.- Pancreas c.- Gastrico. d.- Esofagico.
  • 113. PREGUNTA No. 313 Cual de las siguientes estructuras es la principal que tienen origen de cáncer de páncreas. RESPUESTA a.- Ductos b.- Islotes. c.- Periampular. d.- Acinos
  • 114. PREGUNTA No. 314 Señale cuál es la técnica quirúrgica indicada, en un paciente con adenocarcinoma ductal de páncreas, situado en la cabeza pancreática y potencialmente resecable: a.- Duodenopancreatectomía cefálica. b.- Pancreatoyeyunostomía lateral. c.- Resección corporo-caudal del páncreas. d.- Gastroyeyunostomía.
  • 115. PREGUNTA No. 315 Ante la palpación de una vesícula distendida e indolora en un paciente ictérico y con síndrome constitucional asociado, ¿qué diagnóstico, de los siguientes, hay que sospechar en primer lugar?: a.- Carcinoma vesicular. b.- Coledocolitiasis. c.- Cólico hepático simple. d.- Carcinoma de cabeza de páncreas
  • 116. PREGUNTA No. 316 Señale lo correcto respecto al carcinoma de páncreas: a.- Afecta preferentemente a mujeres. b.- Responde a la radioterapia en un 30-40% de los casos. c.- Es muy quimiosensible. d.- Cuando presenta dolor es habitualmente inoperable.
  • 117. PREGUNTA No. 317 La duodenopancreatectomía puede estar indicada en los siguientes procesos, EXCEPTO en uno. Señálelo: a.- Tumores de ampolla de Vater. b.- Tumores de la cabeza del páncreas. c.- Pancreatitis crónica localizada en la cabeza del páncreas. d.- Tumores primitivos del retroperitoneo.
  • 118. PREGUNTA No. 318 Un enfermo con ictericia progresiva, indolora (bilirrubina total 8 mg/dl) es diagnosticado de masa periampular con PAAF positiva para células malignas. El estudio de extensión es negativo. ¿Cuál debe ser la opción terapéutica más aconsejable de las siguientes? a.- Drenaje externo, nutrición parenteral y continuar el estudio. b.- Instalación de prótesis expansora transtumoral. c.- Duodenopancreatectomía total. d.- Duodenopancreatectomía cefálica
  • 119. PREGUNTA No. 319 ¿Cuál de las siguientes medidas preoperatorias es la más importante a realizar en un paciente severamente ictérico, por cáncer de páncreas posiblemente resecable, en vistas a una mejor evolución postoperatoria? RESPUESTA a.- Descompresión biliar preoperatoria. b.- Nutrición adecuada preoperatoria. c.- Tratamiento antibiótico preoperatorio. d.- Profilaxis antitrombótica preoperatoria.
  • 120. PREGUNTA No. 320 Los insulomas? a.- Casi siempre requieren muestreo venoso selectivo para su localización. b.- Son más frecuentes en la cabeza del páncreas. c.- Por lo general son benignos. d.- Se tratan con pancreatectomía anatómica.
  • 121. PREGUNTA No. 321 ¿ Cual de los siguientes es el síntoma de presentación más frecuente en pacientes con un somatostatinoma? a.- Colelitiasis. b.- Estreñimiento. c.- Hipoglucemia. d.- Hipocalcemia.
  • 122. PREGUNTA No. 322 ¿ Cuál de los siguientes estudios de imagen es el más exacto para identificar tumores endócrinos del páncreas? a.- Ecografía b.- T.A.C. c.- RM d.- Estudio con metayodobencilguanina
  • 123. PREGUNTA No. 323 ¿Cuál de los siguientes es el tumor endócrino pancreático más frecuente? a.- Gastrinoma. b.- Glucogonoma. c.- Insulinoma. d.- Tumor productor de PIV.
  • 124. PREGUNTA No. 324 ¿Cuál de los hallazgos siguientes en la exploración física de un paciente diabético representa una posibilidad de Glucoganoma? a.- Acantosis nigricans. b.- Dematitis. c.- Manchas café con leche. d.- Efélides axilares.
  • 125. PREGUNTA No. 325 Los síntomas y signos de un paciente con un tumor secretor de polipéptido intestinal vasoactivo (VIP) incluyen todos los siguientes, EXCEPTO: a.- Diarrea que no responde a agentes antidiarreicos. b.- Diarrea que persiste durante el ayuno. c.- Hipopotasemia. d.- Alcalosis metabólica grave.
  • 126. CASO CLINICO Femenino 28 años. Acude por historia de palpitaciones y temblor fino en las manos. Tiene insomnio pero lo atribuye al nacimiento de su bebé hace 5 semanas. Se encuentra lactando. A la EF tiene temblor fino distal, sudoración en palmas de las manos y se aprecia un discreto bocio. Labs: TSH <0.002 (0.8-1.8).
  • 127. PREGUNTA No. 326 Cuál de las siguientes medidas está indicada para establecer el diagnóstico? RESPUESTA a.- Determinación de anticuerpos anti receptor TSH. b.- Administrar I131. c.- VSG. d.- Medir Tiroglobulina.
  • 128. PREGUNTA No. 327 La medida que usted ha elegido le permitirá diferenciar entre: RESPUESTA a.- Tiroidis viral vs. tiroiditis postparto. b.- Tiroiditis postparto vs. enfermedad de graves. c.- Trastorno inmunológico vs. enfermedad de graves. d.- Tiroidistis de hashimoto vs enfermedad de graves.
  • 129. PREGUNTA No. 328 Los principales marcadores de Enfermedad de Graves son: RESPUESTA a.- TSI. b.- Tiroglobulina. c.- Anticuerpos anti TPO. d.- Anticuerpos anti Tiroglubilina.
  • 130. PREGUNTA No. 329 La triada de la enfermedad de graves es: RESPUESTA a.- Oftalmopatía, Dermopatía, Osteopatía b.- Oftalmopatía, Dermopatía, Neuropatía c.- Hipertiroidismo, Oftalmopatía, dermopatía d.- Hipotiroidismo, Oftalmopatía, dermopatía.
  • 131. PREGUNTA No. 330 Todas son afirmaciones ciertas acerca del I 131 excepto: RESPUESTA a.- El embarazo y la lactancia son contraindicaciones para su uso b.- Es una terapia indicada en la Enfermedad de Graves c.- Puede generar hipotiroidismo d.- Genera efectos de forma inmediata.
  • 132. PREGUNTA No. 331 Una determinación baja de T4 libre y TSH elevada hacen diagnóstico de? RESPUESTA a.- Hipertiroidismo secundario b.- Hipertiroidismo primario c.- Hipotiroidismo primario d.- Hipotiroidimo secundario
  • 133. CASO CLINICO Femenino de 19 años de edad el cual acude a consulta presentando dolor en epigastrio, tipo punzante, refiere la madre presentar nausea moderada y falta de apetito, niega vida sexual activa.
  • 134. PREGUNTA No. 332 Cual de las siguientes posibilidades diagnosticas es la mas probable considerando su cuadro clínico. ? RESPUESTA a.- Embarazo Ectópico. b.- Apendicitis. c.- Dismenorrea. d.- Gastritis aguda
  • 135. PREGUNTA No. 333 Tomando en cuenta su diagnostico, cual es el tratamiento de inicio que indica? RESPUESTA a.- Analgésicos y antiespasmodicos. b.- Inhibidores de la bomba de protones. c.- Hidratación y vigilancia. d.- Laparotomía Exploratoria.
  • 136. CASO CLINICO Un paciente de 16 años femenino que acudio a su unidad de medicina familiar refiriendo que 10 horas antes inicio con dolor ubicado en la fosa iliaca derecha acompañado de nausea y diarrea leve, niega fiebre, el dolor era intenso pero súbitamente disminuyo, tomo medicamento sin indicación medica.
  • 137. PREGUNTA No. 334 Cual es su conducta a seguir mas apropiada para este caso? RESPUESTA a.- Alta con cita abierta. b.- Antibiotico y antiespasmodico. c.- QS, BH, RX de abdomen inmediatos. d.- Solicita HGC en sangre.
  • 138. PREGUNTA No. 335 Esta patología se caracteriza por incrementar la presión de su luz intestinal, ya que su capacidad no es mayor a .01 ml. Tomando en cuenta la edad de la paciente su anatomía y a la probable etiología es cual es el agente causal? a) E. Coli. b) Sf Fecalis. c) B. Fragilis. d) E. Histolitica.
  • 139. PREGUNTA No. 336 La paciente ingresada en espera de resultado se observa palida, diaforetica, hipotensa e hipotermica, se observaron leucocitosis de 25,000 considerando su conducta terapéutica y diagnostica cual es la conducta mas adecuada a seguir ahora? RESPUESTA a.- Tomar tiempos de coagulación b.- Solicitar plasma fresco congelado y paquetes eritrocitarios. c.- Doble esquema de antibiótico. d.- Laparotomía exploratoria de urgencia.
  • 140. PREGUNTA No. 337 Cual es la complicación crónica mas frecuente debido a la evolución del caso? RESPUESTA a.- Síndrome de mala absorción. b.- Bridas y adherencias intestinales c.- Trastornos motores de colon d.- Síndrome de intestino irritable.
  • 141. CASO CLINICO Un paciente de 45 años de edad es diagnosticado de esófago de Barret largo (segmento de 6 cm) y sigue tratamiento con inhibidores de la bomba de protones (IBP) a dosis estándar.
  • 142. PREGUNTA No. 338 Cuál de las siguientes afirmaciones es cierta? RESPUESTA a.- En su seguimiento se realizará endoscopia sólo si persisten los síntomas. b.- Mientras se mantenga el tratamiento continuo con IBP se elimina el riesgo de cáncer. c.- Se debe asegurar un control adecuado de la secreción ácida con dosis altas de IBP o preferiblemente ajustando la dosis mediante pHmetría. d.- El seguimiento sólo es necesario si existe displasia de alto grado.
  • 143. PREGUNTA No. 339 Tras realizar una endoscopia digestiva a un paciente de 51 años, se nos informa que el diagnóstico de sospecha es "esófago de Barrett". ¿Cuál es, entre las siguientes, la conducta más adecuada? RESPUESTA a.- El paciente debe ser remitido al servicio de Cirugía para realizar un funduplicatura. b.- La indicación correcta es la esofaguectomía. c.- Lo más adecuado, para tener seguridad diagnóstica completa, es esperar al resultado de la biopsia antes de tomar decisiones. d.- Debemos pautar tratamiento médico con omeprazol de por vida y no aconsejar más revisiones.
  • 144. PREGUNTA No 340 En cuál de las siguientes neoplasias gástricas en estadio incipiente se puede considerar como primera opción la erradicación de H. Pylori con antibioterapia e IBP? RESPUESTA a.- Adenocarcinoma gástrico difuso. b.- Adenocarcinoma gástrico de tipo intestinal. c.- Linfoma MALT gástrico. d.- Linfoma alto grado gástrico.
  • 145. PREGUNTA No. 341 Cuál de los siguientes factores NO se considera factor de riesgo de desarrollo del cáncer gástrico? RESPUESTA a.- Anemia microcítica hipocromica. b.- Gastritis crónica atrófica. c.- Infección por Helicobacter Pylori. d.- Enfermedad de Ménétrier.
  • 146. PREGUNTA No. 342 Señalar en relación con el linfoma gástrico primario, cuál es la respuesta acertada? RESPUESTA a.-.- El linfoma gástrico primario es de peor pronóstico que el adenocarcinoma. b.- Los pacientes que responden a la antibioterapia en linfoma MALT deben controlarse mediante endoscopia periódica, debido al desconocimiento de si se elimina el clon neoplásico ó simplemente se suprime de forma temporal. c.- La resección gástrica asociada a quimioterapia permite supervivencias de 80 a 90% a los cinco años en pacientes con linfoma de alto grado localizados. d.- El linfoma gástrico primario es una forma infrecuente de presentación extraganglionar de linfoma.
  • 147. PREGUNTA No. 343 En un paciente de 66 años, diabético no insulinodependiente, al que se le ha realizado una endoscopia, se ha detectado una lesión extensa en el cuerpo gástrico. La biopsia informa adenocarcinoma de tipo intestinal. ¿Cuál de las siguientes afirmaciones es más correcta? RESPUESTA a.- Unos valores de CEA, CA 19.9 y CA 72.4 en el rango de la normalidad nos excluyen la presencia de enfermedad diseminada. b.- Es obligatorio realizar un tránsito gastrointestinal para verificar la extensión c.- El siguiente paso es la realización de ecografía endoscopica y TAC abdominal pélvico. d.- Solicitaremos una ecografía endoscópica para descartar la presencia de metástasis hepáticas y pancreáticas.
  • 148. PREGUNTA No. 344 De entre las siguientes afirmaciones sobre el carcinoma gástrico precoz, señale cuál es FALSA: a.- Operado tiene buen pronóstico, con supervivencia hasta del 95% de los casos a los cinco años. b.- No sobrepasa la submucosa. c.- No tiene capacidad para producir metástasis ganglionares linfáticas. d.- Existe una clara correlación entre la profundidad de la invasión del tumor y la tasa de supervivencia.
  • 149. PREGUNTA No. 345 Cuál de las siguientes entidades NO se asocia con mayor riesgo de cáncer gástrico que el de la población general? RESPUESTA a.- Reflujo duodenogástrico. b.- Enfermedad de Menetrier. c.- Anemia perniciosa. d.- Acalasia.
  • 150. PREGUNTA No. 346 El estudio de mayor especificidad y sensibilidad para hacer el diagnóstico de reflujo gastroesofágico es la: RESPUESTA a.- Endoscopia. b.- Manometría. c.- Medición del ph. d.- Serie esofago-gastroduodenal.
  • 151. PREGUNTA No. 347 Paciente de 68 años que consulta por pirosis. disfágía, leve ocasional y episodios compatibles con regurgitación nocturna hace 2 semanas. Refiere desde hace unos 3 años temporadas anteriores de pirosis y regurgitación. La endoscopia alta practicada demuestra una esofagitis erosiva grave. ¿Qué tratamiento farmacologico, entre los siguientes, es el más adecuado? RESPUESTA a.- Inhibidores de la bomba de protones. b.- Antagonistas de los receptores H2. c.- Sucralfato. d.- Tratamiento combinado con anti-H2 y sucralfato.
  • 152. PREGUNTA No. 347 Varón de 42 años, obeso reciente, no fumador, con asma desde hace 1 año, que empeora después de las comidas y se despierta por las noches con tos irritativa. Sigue tratamiento con broncodilatadores y prednisona. ¿Cuál sería la mejor prueba diagnóstica para aclarar el cuadro? RESPUESTA a.- Endoscopia bronquial. b.- Pruebas de provocación con alergenos. c.- pH esofágico durante 24 horas. d.- Ecodoppler de venas de piernas.
  • 153. PREGUNTA No. 348 ¿Cuál de las siguientes medidas NO forma parte del primer paso del tratamiento del reflujo gastroesofágico? RESPUESTA a.- Elevar la cabecera de la cama. b.- Evitar el tabaco y el alcohol. c.- Reducir las grasas de la dieta. d.- Administrar ranitidina, 150 mg cada 12 horas.
  • 154. PREGUNTA No. 349 En un paciente con esofagitis secundaria a reflujo gastroesofágico, ¿cuál de los siguientes medicamentos deberán evitarse? RESPUESTA a.- Teofilinas. b.- Cisapride. c.- Domperidona. d.- Furosemida.
  • 155. PREGUNTA No. 350 Un paciente de 32 años exhibe una historia de disfagia, de varios años de evolución, que ha progresado en las últimas semanas hasta tener dificultades, no diarias, para ingerir líquidos. Describe también episodios aislados de regurgitación no acida de alimentos y ha tenido, el año pasado, dos episodios de neumonía. La exploración que más probablemente haga el diagnóstico correcto será: RESPUESTA a.- Endoscopia digestiva alta. b.- Radiología esofagogástrica con bario. c.- Estudio de vaciamiento esofágico con radioisótopos. d.- Manometría esofágica.
  • 156. RESPUESTA No. 351 El reflujo gastroesofágico puede relacionarse con cualquiera de las entidades siguientes EXCEPTO una. Señálela: RESPUESTA a.- Asma por reflujo b.- Broncoespasmo. c.- Neumonía recurrente. d.- Sinusitis.
  • 157. PREGUNTA No. 352 Un paciente de 42 años de edad consulta por molestias faríngeas y cierta afonía de dos meses de evolución. NO es fumador ,ni tiene una profesión en que deba forzar la voz. Una exploración laríngea revela un ligero edema delas cuerdas vocales y un ligero eritema de la región interaritenoidea. Interrogado el paciente no refiere pirosis ni regurgitación ácida. ¿Cuál de las siguientes afirmaciones es cierta?: RESPUESTA a.-La presencia de mínimas lesiones laríngeas indican que muy probablemente el paciente tenga también lesiones de esofagitis y por tanto hay que indicar una endoscopia digestiva alta. b.-En ausencia de síntomas de broncoaspiración (tos y sibilancias), las molestias laríngeas no pueden ser atribuidas a enfermedad por reflujo gastroesofágico. c.- La ausencia de síntomas de reflujo (pirosis y regurgitación)no descarta la enfermedad por reflujo. d.-Puede averiguarse si la causa de los síntomas es una enfermedad por reflujo gastroesofágico con una prueba corta administrando ranitidina 150 mg al día durante dos semanas.
  • 158. PREGUNTA No. 353 El objetivo de la cirugía antirreflujo en los pacientes que presentan enfermedad por reflujo gastroesofágico es? RESPUESTA a.- Mejorar el vaciamiento esofágico. b.- Restablecer la función del esfínter esofágico superior. c.- Mejorar la contractilidad esofágica. d.- Mejorar la función mecanica del cardias.
  • 159. CLINIC CASE Female patient is 67 years old, who comes in for tiredness, fatigue, shortness of breath, pain toraccic decreases with rest, dyspnea on moderate exertion, edema of the lower 2/3 of the leg, the physical examination, pallor of skin and mucous membranes, delayed capillary refill, mild rales, gallop and trill presence. The patient is hypertensive for 20 years with moderate adherence to treatment with captopril 50 mg / day, hydrochlorothiazide 25 mg/12 hrs. He has presented two hypertensive crisis, as well as an event of mesenteric insufficiency, laboratories EGO proteinuria, glucosuria, urate, decreased urine density, cholesterol 289 mg / dl, triglycerides 720 mg / dl, creatinine 6.2 mg / dl, BUM 29, kalemia of 5.3, vital signs were BP 160/105 mmHg, HR 96, FR 28, BMI 32.
  • 160. QUESTION No. 354 Echocardiography has noted a FE <40% during admission, the patient presented urinary volume of 20 ml / hr. Considering the current characteristics which functional class is presenting the case? ANSWER a. - Functional Class I. b. - Functional Class II. c. - Functional Class III. d. - Functional Class IV.
  • 161. QUESTION No. 355 Echocardiogram reporting an EF> 45%, premature ventricular electrocardiogram reports, conduct which is the most appropriate drug to decrease ventricular remodeling and improve cardiac electrical activity? ANSWER a. - Sign beta blocker. b. - Start verapamil. c. - Start losartan. d. - Start diltiazem.
  • 162. QUESTION No. 356 Considering the current characteristics which is the K/ DOQI presented by the patient, which is the most suitable method for determining the stage of CKD? ANSWER a. - Creatinine clearance of 24 hrs. b. - Renal biopsy. c. - Renal ultrasonography. d. - Escretora urography.
  • 163. QUESTION No. 357 What is the most appropriate pharmacological behavior considering the K / DOQI and functional class? ANSWER a. - Verapamil. b. - Diltiazem. c. - Telmisartan. d. - Isosorbide.
  • 164. CLINIC CASE Male is 59 years old, admitted to the emergency department because of shortness of breath, tiredness, fatigue, productive cough for 15 days, the night before the current admission, has a history of COPD diagnosed 10 years ago, after smoking (a pack a day), with treatment iprapropio bromide, salbutamol and betamethasone, and was diagnosed as hypertensive two years ago, starting with lifestyle changes and sodium-restricted diet, consider a year ago it was changed to include captopril three months later by cough increased, from entoces take losartan, hydralazine. The EF is observed with conjunctival hyperemia plethoric facies, moderately cyanotic oral mucosa, a murmur is heard right carotid, jugular venous distension grade II, rales subcrepitantes bibasilar, tipanica resonance to percussion, slightly diminished heart sounds, rhythmic, abdomen with hepatomegaly 5 cm below the costal margin.
  • 165. QUESTION No. 358 Considering the current state of the patient which is the most appropriate behavior to follow to establish cardiopulmonary function? ANSWER a. - Spirometry. b. - Echocardiogram. c. - Value of natriuretic. d. - Chest radiograph.
  • 166. QUESTION No. 359 After previous testing had an EF less than 40%, the chest radiograph showed an index cardiotoraccico pathological bilateral cotton wool spots, increased brocovascular frame, gas PCO 49, PO2 85, potassium 6.3, natriuretic factor decreased to physical expxloracion crackles are heard, wheezing moderate hyperresonance to percussion, galloping apresiar is reached, which is the most appropriate behavior to continue to decrease preload, considering all factors? ANSWER a. - Remove potassium sparing diuretic loop diuretic. b. - Start with dihydropyridine calcium antagonist quick action. c. - Increased peripheral vascular capacitance nitrites. d. - Water restriction, sodium-free diet, loop diuretic.
  • 167. CLINIC CASE Male is 68 years old which is diagnosed and treated for typical angina 5 years, however 6 months from the date of the chest discomfort was moderate and sometimes at rest, comes to emergency because oppression has accompanied toracicca left shoulder pain that radiates to the arm, which was added when he began discussing the inconvenience, did not yield to rest and hard over the usual time, has a history of high blood pressure from 20 years with current treatment with captopril 25 mg every 12 hrs, hydralazine 25 mg / 12 hrs, 5 years ago was added propranolol, pravastatin and aspirin to manage your initially stable angina, your income is observed diaphoretic, with paleness , anxious, with distal cyanosis, his vital signs were BP 105/70 mmHg, HR 101 bpm, RR 29 rpm.
  • 168. QUESTION No. 360 Considering the clinical diagnosis which is more likely at this point? ANSWER a. - Decompensated typical angina. b. - Initial atypical angina. c. - Acute myocardial infarction. d. - Angina heart failure.
  • 169. QUESTION No. 361 Which of the following is most useful diagnostic aids at this time to maintain an adequate clinical behavior? ANSWER a. - Search biomarkers of ischemia. b. - Perform electrocardiogram. c. - Coronary angiography. d. - Programming dobutamine stress test.
  • 170. QUESTION No. 362 What is your class with the observed clinical functional? ANSWER a. - Class I. b. - Class II. c. - Class III. c. - Class IV
  • 171. QUESTION No. 363 The following studies were performed to the patient what is your opinion on the X-ray and ECG? ANSWER a. - Facts pulmonary edema. b. - Facts heart failure. c. - AMI data. d. - Facts COPD.
  • 172.
  • 173.
  • 174. QUESTION No. 364 What is the most important immediate behavior to continue? ANSWER a. - Morphine, oxygen, nitrite and antiplatelets. b. - Diuretic, morphine, verapamil and oxygen. c. - Diuretic, antiaginoso and thrombolysis. d. - Morphine, nitrite, thrombolysis, enoxaparin.
  • 175. CLINIC CASE Turn to the outpatient female patient aged 25, comes because it does not produce enough milk to breastfeed has 8 days postpartum, denies other symptoms, he explains the technique for lactation, however the observed increase exploration symmetrical thyroid gland not palpable nodules or painful areas, not identify voice changes, the patient is originally from Chiapas.
  • 176. QUESTION No. 365 What the most appropriate action to take in case? ANSWER a. - Conduct assessment of HRT and T4. b. - Indicate iodine supplementation. c. - Indicate diet rich in iodine, cites six months. d. - Request antithyroid antibody profile.
  • 177. QUESTION No. 366 6 months later was sent to endocrinology because it showed increased weight remained the same characteristics of volume and consistency of the thyroid gland, thyroid profile was made ​with the following results low total T4, normal TSH and indicated 100 mcg / day considering the evolution, which of the following differential diagnosis should nonthyroidal causes. ANSWER a. - Low T4 syndrome. b. - Nontoxic multinodular goiter. c. - Toxic multinodular goiter. d. - Hyperfunctioning nodule.
  • 178. CLINIC CASE Subclinical hypothyroidism patient diagnosed and treated properly with oral thyroxine. A few months later, he developed bilateral exophthalmos and simultaneously presents serum antibodies anti-TSH receptor positive. Then adjourned thyroxine treatment, and it is found that hypothyroidism has progressed to become primary. A diagnosis of hypothyroid Graves' disease.
  • 179. QUESTION No. 367 What is the most frequent complication in the long term for the patient due to treatment should be instituted. ANSWER a. - Osteoporosis. b. - Arteriosclerosis. c. - Pulmonary edema. d. - Dyslipidemia.
  • 180. CLINIC CASE Attend annual review consultation female patient of 31 years, which has the diagnosis of simple goiter, presented auscencia refers to menstrual periods on 4 occasions in the year, in addition to presenting upper airway infections on two occasions, has presented fatigue, generalized weakness, weight gain. Physical examination shows bradypsychia and bradilalica, looks dry and rough. Vital signs were BP 110/70 mmHg.
  • 181. QUESTION No. 368 The patient was admitted because of malaise and hypothermia, which is the immediate conduct to follow to establish a presumptive diagnosis. ANSWER a. - BH, QS, EGO. b. - Request TSH, T4. c. - Apply thyroid peroxidase antibodies. d. - Thyroid biopsy.
  • 182. QUESTION No. 369 The following results were obtained hematocrit 32%, hemoglobin 10.1, WBC 5,900, 219 Central glucose, cholesterol 310. Considering the results which is the most appropriate action to take. ANSWER a. - Start levothyroxine 150 mcg / day. b. - Apply thyroid antibody profile. c. - Stabilization and shipping back to the second level. d. - Administration and prednisone 20 mg levothyroxine 100 mcg / day.
  • 183. QUESTION No. 370 2 months after the patient returns to the first level control of attention in its counter-note is the diagnosis of Hashimoto's thyroiditis, considering the evolution of the patient, which of the following is least likely comorbidities. ANSWER a. - Diabetes mellitus. b. - Addison's Disease. c. - Pernicious anemia. d. - Pituitary leukocyte infiltration.
  • 184. CLINIC CASE Girl, 12 years old, who for 4-5 months started with nervousness, tachycardia, heat intolerance, sweating and impulsivity to eat, no weight loss or insomnia. Also refers bulge in the front of the neck, without pain or other signs of inflammation associated. His mother has hypothyroidism treated with L-thyroxine for 7 years. In Physical examination revealed bilateral ocular protrusion and a grade II goiter, elastic consistency, without pain or other signs of inflammation. The study showed hyperthyroidism with thyroid hormones elevated thyroid antibodies. Thyroid ultrasonography objectified both lobes enlarged (right 18 x 18.8 x 53mm, left 15.9 x 18 x 51mm), heterogeneous, hypoechoic areas with multiple indistinct borders that shaped and marked with Doppler hypervascularity.
  • 185. QUESTION No. 371 What is the most appropriate therapeutic measure long term? ANSWER a. - Partial thyroidectomy. b. - Radioactive iodine. c. - Propylthiouracil. d. - Methimazole.
  • 186. CLINIC CASE This male patient of 60 minutes RN, obtained by caesarean iteratively presenting other fetal distress, by intrauterine activity decreased and FC decreases up to 110 bpm is observed with very fine movement at the beginning and shortly presented mainly mild cyanosis periungeal and oral, oral secretions are abundant apresiaron, physical examination normal bilateral rales are heard, little audible peristalsis and temperature of 36.1 degree, apresió tinted secretions expelled meconium during scanning.
  • 187. QUESTION No. 372 Considering the clinical picture what their conduct to follow? ANSWER a. - Keep the patient under observation continuously. b. - Search psobles hidden pathologies. c. - Perform chest radiography. d. - No pathological data are adaptive.
  • 188. QUESTION No. 373 What is the therapeutic less appropriate at this time of the case? ANSWER a. - Hold fast to identify cause. b. - Place in neutral environment. c. - Administer oxygen by headbox 40%. d. – Gas analysis and pulse oximetry.
  • 189. QUESTION No. 374 Which of the following is not normal blood gas parameters in the case expect? ANSWER a. - pH <7.34. b. - PaCO> 45 mmHg. c. - HCO3> 40mEq / L. d. - SpO2 <95.
  • 190. CASO CLINICO Se ingresa a paciente masculino de 45 años de edad el cual cuenta con antecedente de ser portador de VIH positivo con cifras de CD4 de 85, actualmente suspendió el tratamiento antirretroviral, el motivo del ingreso es debido a dolor en la región lumbar intenso el cual mostro datos de ostemielitis, el paciente cuenta con el antecedente de presencia de tos productiva con esputo amarillento y sanguinolento, perdida de peso importante, por lo que previamente se realizo baciloscopia siendo positiva para tuberculosis, apoyada por los parámetro radiográficos.
  • 191. PREGUNTA No. 375 Cual es el tratamiento inicial mas adecuado. RESPUESTA a.- Isoniazida, rifampicina, etambutol y pirazinamida. b.- Isoniazida, rifampicina, etambutol y pirazinamida mas TRV. c.- Isoniazida, rifampicina, etambutol, pirazinamida y estreptomicina. d.- Isoniazida, rifampicina y etambutol.
  • 192. CASO CLINICO Paciente de 56 años, sexo femenino, con antecedente de dermatomiositis, en tratamiento con hidroxicloroquina 200 mg, metotrexato 10 mg y prednisona 30 mg diarios. Fue hospitalizada por cuadro de 10 días de evolución, con fiebre, compromiso del estado general y pérdida de peso, encontrándose crepitaciones bilaterales al examen pulmonar. La TAC de tórax mostró un infiltrado nodular bilateral, extenso, con cavitaciones biapicales. La baciloscopia obtenida por lavado bronquio-alveolar (LBA) resultó positiva y la reacción de polimerasa en cadena (RPC) positiva para Mycobacterium tuberculosis. Además úlceras en los pulgares aparecidas cuatro semanas previas al ingreso. Al examen físico se observó una úlcera de bordes bien delimitados con fondo sucio y bordes necróticos, no dolorosa, ubicada en el pulpejo del dedo pulgar izquierdo. S. aureus y Candidaparapsilosis. La tinción de Ziehl Neelsen en tejido fue positiva.
  • 193. PREGUNTA No. 376 Cual es el origen mas probable que cause las ulceras en pulgares? RESPUESTA a.- Farmacologica. b.- Bacteriana. c.- Autoinmune. d.- Mixta.
  • 194. CASO CLINICO Se trata de masculino de 19 años de edad ingresado al servicio de urgencias presentando fiebre, cefalea intensa, dolos abdominal difuso pero ligero, pero con mialgias intensas, al examen físico se observa con 38,5 c, FC 112, TA 120/70 su estado general es adecuado en crecimiento y desarrollo con diaforesis, se mantiene alerta y orientado, se realizan estudios de laboratorio y gabinete de rutina donde se observa únicamente plaquetas con 84,000, se realiza puncion lumbar donde se observa auscencia de eritrocitos, proteínas y glucosa normales, solo se observa 5 monocitos. Como antecedente de importancia refiere regresar de un campamento y realizar escalado de montañas, actualmente continua en el servicio de medicina interna.
  • 195. PREGUNTA No. 377 Considerando los hallazgos así como los antecedentes usted integra un diagnostico, y debe iniciar tratamiento, cual es el más adecuado? RESPUESTA a.- Rifampicina. b.- Doxiciclina. c.- Tetraciclina. d.- Ceftriaxona.
  • 196. PREGUNTA No. 378 Cual de las siguientes contraindicaciones son menos probables de la farmacoterapia elegida? RESPUESTA a.- Incremento de los efectos tóxicos de alcaloides ergotamínicos y metotrexato. b.- Disminuir la eficacia de los anticonceptivos orales. c.- El uso concomitante con metoxiflurano puede producir toxicidad renal fatal. d.- Disminuye los efectos tóxicos de ciclosporina A.
  • 197. CASO CLINICO Ingresa paciente masculino de 48 años de edad, ingeniero que acaba de regresar de convención en medio oriente, presenta fiebre, diaforesis, ataque al estado generalizado, fatiga y adinamia, así como obnubilación, a la exploración se observa hepato y esplegnomegalia leve a moderada, es ingresado para realizar estudios encontrando paludismo por plasmodium falciparum, parasitemia de 7 %, hematocrito de 21 %, bilirrubina de 7.8 mg/100 ml, creatinina 2.6 mg/100 ml. Se ingresa posteriormente a cuidados intensivos, se realiza asistencia ventilatoria y se indica neuroproteccion debido a la gravedad del cuadro.
  • 198. PREGUNTA No. 379 Cual es el mejor esquema de tratamiento para este caso o prevenirlo? RESPUESTA a.- Quinina intravenosa. b.- Quinidina Intravenosa. c.- Artesunato Intravenoso. d.- Cloroquina.
  • 199. PREGUNTA No. 380 Considerando los efectos adversos, cual de los siguientes casos es menos probable contraindicarlo? RESPUESTA a.- Paciente psicótico. b.- Paciente cardiópata. c.- Paciente nefropata. d.- Paciente hepatopata.
  • 200.
  • 201. PREGUNTA No. 381 Considerando el frotis cual de las siguientes formas se encuentran presentes? RESPUESTA a.- Trofozoitos. b.- Eritrozoitos. c.- Quistocitos. d.- Gametozoitos
  • 202. PREGUNTA No. 382 Cual de las siguientes especies es el huésped original del agente etiológico del caso? RESPUESTA a.- Gorilas. b.- Humanos. c.- Moscos. d.- Vacas
  • 203. CASO CLINICO Se trata de paciente masculino de 57 años de edad el cual acude al servicio de urgencias debido a tos intensa no productiva y fiebre, el paciente cuenta con los siguientes antecedentes, originario de guanajuato, empleado de la construcción, alcoholismo y tabaquismo positivo desde hace 40 años, actualmente bajo tratamiento por EPOC y cirrosis hepática, se realizan estudio de laboratorio y gabinete de rutina donde se observa un reporte con disminución de la respuesta inmunológica esperada, la radiografia de torax presenta un patrón característico, sin embargo el cuadro respiratorio bajo se ha intensificado por lo que se traslada a terapia intensiva, donde se reporta positivo para legionella.
  • 204. PREGUNTA No. 383 Cual es tratamiento antibiótico más apropiado para este caso? RESPUESTA a.- Azitromicina. b.- Levofloxacina. c.- Trimetoprim/sulfametoxazol. d.- Penicilina.
  • 205. PREGUNTA No. 384 Considerando el agente etiológico, cual de las siguientes fuentes de contagio no es relevante para adquirirlo? RESPUESTA a.- Sistema de aguas de grandes edificios. b.- Sistemas de agua de hoteles y hospitales. c.- Maquinas de rocio, spas y fuentes de agua termal. d.- Sistemas de aire acondicionado.
  • 206. PREGUNTA No. 385 Cual de los siguientes medios auxiliares no es util para su diagnostico? RESPUESTA a.- Hibridación con sondas moleculares específicas. b.- Cultivo en medio BCYE-alpha Agar + Antibióticos. c.- Positivo en las pruebas de la catalasa y oxidasa. d.- Medios con sangre (Agar sangre o Agar chocolate).
  • 207. CASO CLINICO Se trata de paciente femenino de 38 años de edad la cual acude al servicio de urgencias debido a que presenta fiebre, cefalea y dolor en la región de la nuca, como antecedentes de importancia la paciente trabaja en un aviario, no es diabética ni hipertensa, aparentemente sana antes de este episodio, a la exploración física se observa conciente, irritable, orientada, facies algicas, se realiza TAC de cabeza el cual no revela datos de importancia, se realiza punción lumbar reportándose presión de apertura de 20 cmH2O, leucocitos 15 células (90 % monocitos), proteínas de 0.6 g/L, glucosa de 50mg/dl y tinción positiva con tinta china.
  • 208. PREGUNTA No. 386 Cual es el tratamiento mas adecuado para el caso? RESPUESTA a.- Anfotericina. b.- Fluconazol. c.- Cexfriaxona. d.- Vancomicina.
  • 209. PREGUNTA No. 387 Cual de las siguientes manifestaciones es menos probable en esta patología? RESPUESTA a.- Meningoencefalitis, con atrofia cortical. b.- Presentan cuadros psiquiátricos. c.- Durante muchas semanas no da síntomas. d.- Difícilmente presentan fiebre.
  • 210. PREGUNTA No. 388 Cual de los siguientes agentes diferenciales es menos frecuente? RESPUESTA a.- Histoplasmosis. b.- Aspergilosis. c.- Candidiasis. d.- Toxoplamosis
  • 211. PREGUNTA No. 389 Cual de las características del mecanismo de acción del tratamiento elegido, no se presenta? RESPUESTA a.- Salida de iones sodio. b.- Salida de iones cloro. c.- Salida de iones potasio. d.- Salida de hidrogeniones.
  • 212. PREGUNTA No. 390 Cual de las siguientes manifestaciones adversas es menos frecuente de observarse? RESPUESTA a.- Aumento de fosfatasa alcalina, aumento de urea, anomalías en pruebas de función hepática. b.- Taquicardia supraventricular, bradicardia, fibrilación auricular, bloqueo auriculoventricular de segundo grado y extrasístoles ventriculares. c.- Acidosis, alteraciones de potasio, calcio, cloruros, fosfato, alteraciones del equilibrio hídrico, hiperbilirrubinemia, hipopotasemia, hipomagnes emia. d.- Hipertensión, hipotensión, astenia, edema generalizado.
  • 213. CASO CLINICO Se trata de masculino de 34 años de edad el cual tiene catéter para hemodiálisis, actualmente con renopatia terminal, acude a consulta debido a que presenta fiebre y dolor intenso en la región baja de la espalda, a la exploración física se observa ansioso, irritable, diaforético así como ataque al estado generalizado sin embargo signos vitales 120/80, FC 89, FR 21, temperatura 38.7 C, el sitio donde se encuentra colocado el catéter se observa rojo y caliente, no se observan secreciones ni olor, la biometría hemática reporta 16 700 con 13 % de bandas, se realiza IRM donde se reporta lesiones compatibles con osteomielitis vertebral.
  • 214. PREGUNTA No. 391 Cual es agente etiológico más probable? RESPUESTA a.- Streptococcos. b.- Stafilococcos. c.- Mycoplasmas. d.- Tuberculosis.
  • 215. PREGUNTA No. 392 Cual de los siguientes agentes es el mas virulento? RESPUESTA a.- Staphylococcus aureus b.- Staphylococcus epidermidis, c.- Staphylococcus saprophyticus d.- Staphylococcus haemolyticus.
  • 216. PREGUNTA No. 393 Cual de los siguientes no son factores de virulencia y sirven además de evasores de la fagocitosis para la especie diagnosticada? RESPUESTA a.- La presencia de catalasa. b.- La presencia de coagulasa. c.- La fermentación del azúcar manitol. d.- Presencia de B lactamasa.
  • 217. PREGUNTA No. 394 Debido a la producción de toxinas, cual de las siguientes manifestaciones es menos frecuente? RESPUESTA a.- Diarreas, vómito, náuseas. b.- Separa el estrato granuloso del córneo. c.- Forúnculosis asepticos. d.- Impétigo ampolloso.
  • 218. CASO CLINICO Femenino de 60 años la cual trabaja como intendencia en una veterinaria, refiere que hace 10 días inicio con fiebre, escalofríos, cefalea, nauseas, dolor muscular, se automedica con fármacos sintomáticos, mejorando su estado general sin embargo regresan los síntomas y agregandoce ictericia, a la exploración clínica se presenta 38.9 C temperatura, FC 1010, TA 140/90 mmHg, la saturación por oximetro 92 %, a la percusión se presenta dolor hepático, se percibe leve crecimiento, sin esplenomegalia, los estudios de laboratorio BUN de 64, creatinina 3.6, bilirrubina de 64, AST 86, ALT 103, fosfatasa alcalina de 390, leucocitos 11 000, 13% de bandas y 80 % polimorfonucleares, hematocrito 33% y plaquetas de 145, punción lumbar revela pleocitosis, TAC de torax muestra inflitrados en flama difusos compatibles con hemorragia pulmonar. (ver imagen)
  • 219.
  • 220. PREGUNTA No. 395 Cual es diagnostico más probable? RESPUESTA a.- Neumonitis intersticial aguda. b.- Leucemia mieloide aguda. c.- Fiebre por Streptobacillus moniliformis. d.- Infección por Leptospira interrogans.
  • 221. PREGUNTA No. 396 Considerando el agente etiológico, cual de los siguientes animales es el menos transmisor? RESPUESTA a.- Perros. b.- Ratas. c.- Gatos. d.- Humanos.
  • 222. PREGUNTA No. 397 Considerando la patogenia del padecimiento, cual de la siguiente descripción no es habitual? RESPUESTA a.- La bacteria entra al organismo por la piel o mucosas. b.- Se disemina por la sangre atacando diversos órganos. c.- Se replica en el hígado y riñones. d.- Se elimina por la orina, para transmisión.
  • 223. PREGUNTA No. 398 Considerando el cuadro clínico, cual de los siguientes diagnósticos diferenciales es menos frecuente? RESPUESTA a.- Dengue. b.- Fiebre amarilla. c.- Malaria. d.- Influenza.
  • 224. CASO CLINICO Femenino de 23 años de edad, acude a consulta refiriendo fiebre, escalofríos así como dolor de espalda y malestar generalizado. Cuenta con antecedente de trastorno bipolar y abuso de sustancias, actualmente bajo tratamiento sin embargo con recaídas. A la exploración física se observa lesiones tipo punción en pliegue cutáneo en antebrazo por probable inyección intravenosa de sustancias, al parecer de drogas sin aceptarlo la paciente, los signos vitales son 38.5 C de temperatura, FC 110 lpm, FR 24, tensión arterial de 110/60 mmHg, Rx de tórax con varios infiltrados nodulares periféricos con cavitaciones.
  • 225. PREGUNTA No. 399 Cuál es la patología más grave y mas probable que la paciente presenta? RESPUESTA a.- Neumonia b.- Endocarditis. c.- Septicemia. d.- CID.
  • 226. CASE REPORT These male patient aged 49 who comes to the office because it has a palpable mass in the mandibular angle, the patient has a history of major importance such as alcoholism (daily) for 10 years, mostly bad health habits mouth, overweight and glucose intolerant untreated, the patient reported that the mass which has grown slowly comes, occasionally pressing the drains, which presents yellowish discharge, exploration shows an inflamed area of 6 x 6 cm, with presence of yellowish secretion, is soft and indurated to the periphery and slightly tender to palpation, a sample for culture.
  • 227. QUESTION No. 400 What is the agent most likely considering the clinical case. ANSWER a. - Actinomyces. b. - Mucormycosis. c. - Nocardia. d. - Peptostreptococcus.
  • 228. CASE REPORT These male patient aged 19 who is admitted to hospital because his house was found confused, disoriented with fever, relatives refer the patient often infectious types of upper airway, with no treatment usually are completed, a physical examination is widespread in poor, diaphoretic, febrile with a stiff neck, and equally without compromise cardiopulmonary abdomen, lumbar puncture is performed where leukocytes reports of 1800, 95% neutrophils, glucose 35 mg / dl and 100 mg protein / dl.
  • 229. QUESTION No. 401 What is the most appropriate antibiotic pharmacological approach. ANSWER a. - Ampicillin and vancomycin. b. - Ampicillin and gentamicin. c. - cefotaxime and doxycycline. d. - cefotaxime and vancomycin.
  • 230. CASE REPORT Female patient is 70 years old which she consults because it has for several months, nonproductive cough, fatigue, weight loss, the patient has a history of tuberculosis over 20 years ago which was treated be able to obtain sputum for culture as well as blood cultures, lab reports reported presence of acid-fast bacilli Mycobacterium avium complex indicating, the chest radiograph reports bronchiectasis and small nodules scattered throughout the lung parenchyma. The patient has no other important background for the clinical picture.
  • 231. QUESTION No. 402 What is the best treatment scheme? ANSWER a. - Clarithromycin and ethambutol. b. - Clarithromycin and rifampicin. c. - Levofloxacin and rifampin. d. - Prizinamida, Isoniazid, rifampicin and ethambutol.
  • 232. QUESTION No. 403 Which of the following means of laboratory diagnosis is not useful for this entity? ANSWER a. - (Lowenstein-Jensen). b. - (Truant). c. - (Middle-brook). d. - (Ziehl-Neelsen and Kinyoun).
  • 233. CASE REPORT This is a woman, 21 years old, who presented an acute febrile 39-40 ° C, followed on successive days of a generalized itchy rash. After your health in a pediatric center of its region, was diagnosed clinically and serologically classical dengue and treated symptomatically with paracetamol. A week later, once the acute phase symptoms were in an almost complete regression phase, began experiencing slurred speech and to wander, dysphagia, progressive muscle weakness in all four limbs and decreased level of consciousness.
  • 234. QUESTION No. 404 What is the most appropriate diagnostic behavior to identify the cause of the complications? ANSWER a. - Proof of thick b. - Peripheral smear c. - Liver function tests. d. - Biometia hematic.
  • 235. QUESTION No. 405 Which of the following statements is most useful for prognosis of the case? ANSWER a.- Fever of sudden onset. b.- Headache c.- Joint pain d.- Muscle pain
  • 236. CASO CLINICO Se trata de masculino de 54 años de edad el cual cursa con cirrosis alcoholica e hipertensión portal asi como varices esofágicas y encefalopatía hepática, se encuentra ingresado en medicina interna, debido a que inicio hace tres días con alteración del estado de alerta, a la exploración se encuentra con temperatura de 38.4 C, FC 110, FR 14, TA 100/60, continua con disminución del estado de alerta, se observa asterixis leve, su estado cardiopulomnar no reporta datos agudos, el abdomen se encuentra distendido y tenso pero no hay hipersensibilidad, no hay datos de sangrado de tubo digestivo, la biometría hemática reporta 4,800 leucocitos, hematocrito de 32% plaquetas de 95 000.
  • 237. PREGUNTA No 406 Cual método diagnostico es el mas adecuado para establecer un diagnostico? RESPUESTA a.- Panendoscopia. b.- Paracentesis. c.- Lactulosa. d.- Concentración de amoniaco.
  • 238. PREGUNTA No. 407 Cual de las enfermedades metabólicas congénitas del hígado que conducen a cirrosis hepática es la menos frecuente observar. RESPUESTA a.- Hemocromatosis primaria. b.- Enfermedad de wilson. c.- Deficiencia de alfa-1 antitripsina. d.- Esteatohepatitis asociada a diabetes y dislipidemia.
  • 239. PREGUNTA No. 408 El paciente presenta insuficiencia renal prerrenal funcional, que condición del SHR es menos probable observar? RESPUESTA a.- Reversible sin que exista alteración de la estructura renal. b.- Se produce debido a una intensa vasoconstricción renal c.- Tipo I con pronostico fatal a corto plazo. d.- Tipo III con forma más insidiosa, con un pronóstico ligeramente mejor.
  • 240. PREGUNTA No. 409 Se reportaron los siguientes datos, albumina 2.1, bilirrubina total 35, INR 12, según la clasificación de Child-Pugh, en que estado se encuentra el paciente? RESPUESTA a.- A. b.- B c.- C d.- D
  • 241. PREGUNTA No. 410 Cual es su sobrevida del caso a 2 años? RESPUESTA a.- 85 b.- 57 c.- 35 d.- 18
  • 242. CASE REPORT Male is 25 years dedicated to the field, reports that while working was bitten by strange dog, which failed to capture, known cases of rabies in the area in undomesticated animals, comes to the office an hour later the fact.
  • 243.
  • 244. QUESTION No. 411 What is the most appropriate behavior to follow in this case. ANSWER a. - Wash the wound and surveillance. b. - Wash the wound and administration of tetanus. c. - Wash the wound, administration of tetanus immune globulin and implement human. d. - Wash the wound, administer tetanus, applying human immune globulin and human diploid cell vaccine.
  • 245. QUESTION No. 412 In the following structural protein which is the major antigenic component, responsible for the formation of neutralizing antibodies are those that confer immunity? ANSWER a. - Glycoprotein. b. - Matrix proteins. c. - Nucleoprotein. d. - Nucleocapsid.
  • 246. QUESTION No. 413 Which of the following considerations should not be taken into account for the use of immunoglobulins? ANSWER a. - Do not exceed the dose prescribed by your potential interference with the production of antibodies induced by vaccination. b. - Not used in the same site as the vaccine, or reuse the syringe. c. - ot recommended in individuals previously immunized with diploid cells. d. - Gamma globulin provides immediate protection, which lasts about 42 days.
  • 247. CASE REPORT These male patient aged 30 who attends a program of field work carried out in a camp at the end of the day we made a cookout where they served a barbecue, the return of the course presents moderate discomfort beginning to intensify, characterized by fever, headache, malaise, abdominal pain and diarrhea, a physical examination is observed only vague discomfort, laboratory studies reported slightly elevated leukocyte and erythrocyte, Wright staining shows neutrophils, adds that because previously suffered from inflammatory bowel disease was suggested a colonoscopy with biopsy which was conducted where the following was reported, mucosal infiltration by neutrophils, monocytes and eosinophils, with epithelial damage and loss of mucus also glandular degeneration and cryptic abscesses.
  • 248. QUESTION No. 414 Which of the following is the most likely pathogens in this case? ANSWER a. - Escherichia. b. - Norwalk. c. - Campylobacter. d. - Salmonella.
  • 249. QUESTION No. 415 Which of the following observations is less frequently observed agent? ANSWER a. - has an incubation period of 2 to 5 days. b. - is manifested mainly by the appearance of fever, abdominal pain, and diarrhea. c. - can cause reactive arthritis, Guillain-Barré syndrome. d. - interpersonal contact transmission or by contact with infected animals.
  • 250. CASE REPORT Query is presented male age 19 which states that from a previous day begins clinical picture characterized by cramping abdominal pain and watery diarrhea which has increased over the course of hours, is currently a college student and food consumed in the street.
  • 251. QUESTION No. 416 What is the most appropriate treatment at this time of suffering. ANSWER a.- Metronidazole: 250 mg orally 1 capsule c / 8 h for 5 days. b.- Tinidazole 600 mg / kg / day orally for 3 days or 2 grams in a single dose in adults. c.- Furozalidone: 7 mg / kg / day orally for 8 days. d.- Albendazole: 400 mg / orally for five days.
  • 252. QUESTION No. 417 Considering the epidemiology of the disease, which of the following observations is less frequent? ANSWER a. - It is a cosmopolitan parasitosis b. - Predominant in children. c. - They are usually asymptomatic in children. d. - Causes malabsorption and malnutrition.
  • 253. QUESTION No. 418 Which of the following observations about the current illness, is more useful for diagnosis? a. - Direct observation of ovoid cysts. b. - A negative test excludes infection. c. - The method coproparasitoscopic by Faust. d. - duodenal biopsy.
  • 254. QUESTION No. 419 Which of the following observations about the current illness, is more useful for diagnosis? a. - Direct observation of ovoid cysts. b. - A negative test excludes infection. c. - The method coproparasitoscopic by Faust. d. - duodenal biopsy
  • 255. QUESTION No. 420 Which of the following adverse drug reactions in the most frequently chosen? ANSWER a. - Nausea, diarrhea and a metallic taste in the mouth. b. - Has been accompanied by thrombophlebitis. c. - Headache, dizziness, vomiting, glossitis, stomatitis, p aresthesia or dark colored urine or reddish brown. d. - Emergence of a black tongue, leukopenia, neutropenia.
  • 256. CASE REPORT Male 51 years old who starts for 3 days with productive cough, generalized state attack, fever, shortness of breath, chest film in compliant data can be seen bottom right lobar pneumonia and a 1.5-cm round nodule upper lobe left untreated presenting improvement graduating in a week, outpatient CT scan, which shows nodule of 1.5 by 1.8 located in the same position, no calcifications are seen with slightly scalloped edges, mediastinal lymphadenopathy are not observed or pathological data, not appreciated exudates.
  • 257. QUESTION No. 421 What is the most appropriate study to continue the diagnosis in this case? ANSWER a. - MRI b. - FDG PET c. - Bronchoscopy. d. - CT scan at 6 months.
  • 258. QUESTION No. 422 In nonsmokers, the onset of this condition is the result of a combination of factors, which is moreless common in women. ANSWER a. – Genetic. b. - Exposure to radon gas. c. - Air pollution. d. – Secondhand smoke.
  • 259. CASE REPORT Male 71 years old with back pain for two months of sudden onset without precipitating factor, adding that the pain increases and decreases when lying down during the day and with movement. The patient is from the State of Mexico, was a construction worker with alcoholism and smoking positive for over 30 years, which left 15 years ago, not diabetic or hypertensive, the physical examination is observed with adequate general appearance, alert and oriented body mass index of 25, is made of routine laboratories where only performed finding elevated alkaline phosphatase and column radiography lytic lesion is observed only in the L3 vertebra.
  • 260. QUESTION No. 423 The clinical picture indicates a probable malignancy, which is the most likely? ANSWER a. - Pancreatic Cancer. b. - Gastric cancer. c. - Thyroid cancer. d. - Lung cancer.
  • 261. QUESTION No. 424 What is more likely to find such? ANSWER a. - Cell Lung Cancer Small b. - Carcinoid c. - Lung cancer non-small cell d. - Unspecified Lung Cancer
  • 262. QUESTION No. 425 Considering the type of which of the following condition is less common subtypes? ANSWER a. - Papillary adenocarcinoma b. - Adenocarcinoma (not specified) c. - Bronchioloalveolar carcinoma d. - Adenosquamous carcinoma
  • 263. CASE REPORT Male is 73 years old which is from Merida, now lives alone, suffers from diabetes mellitus for 15 years, hypertension, triglycerides, cholesterol and high uric acid, the questioning is unknown dosage for his sufferings, refers indicated that glibenclamide, bezafibrate, enalapril, allopurinol and pravastatin, refers to the family for more than two years have trouble remembering things he is doing, has become suspicious, hidden things to prevent the theft, poverty thought, thought and ideas lock damage and theft.
  • 264. QUESTION No. 426 Due to multiple disease entities in the patient which is most likely neurological complication. ANSWER a. - Vascular dementia is the most likely. b. - Alzheimer's Dementia is the most likely. c. - Prion Dementia is the most likely. c. - Pick's dementia is the most likely.
  • 265. CASE REPORT Male is 65 years old which comes because for four months has asymmetric distal weakness, and muscle spasms and twitches concerns that arise with voluntary movements, physical examination shows weakness of the hands and extensor and difficulty moving the tongue and face, muscle stretch reflexes are observed increased, exploring the sensitivity changes are not observed, the mental examination is ongoing thought and language and appropriate content, humor and affection preserved, other higher mental functions are intact.
  • 266. QUESTION No. 427 What is the most likely clinical diagnosis? ANSWER a. - Amyotrophic lateral sclerosis. b. - MS. c. - Transverse myelitis. d. - Guillain-Barre Syndrome.
  • 267. QUESTION No. 428 Which of the following differential diagnoses is moreless likely? ANSWER a. - Spinal Muscular Atrophy b. - Primary lateral sclerosis. c. - Hawthorn progressive bulbar muscular atrophy. d. - Olivopontocerebellar degeneration.
  • 268. QUESTION No. 429 Which of the following treatments has a function for reducing the symptoms reported? ANSWER a. - Memantine. b. - Rivastigmine. c. - Topiramate. d. - Galantamine
  • 269. CASE REPORT Female is 27 years old which has 22 weeks pregnant, goes to his third prenatal visit, to review the records you see it has provided independent third time a blood pressure of 140/90, you perform a routine test to support his suspected diagnosis of pre-eclampsia, shows lower limb edema, proteinuria, constant headache.
  • 270. QUESTION No. 430 Which of the measures is not appropriate? ANSWER a. - Outpatient management. b. - Hydralazine 50 mg every 12 hrs. c. - hiposodia diet and fluid restriction. d. - Captopril 25 mg every 24 hrs.
  • 271. QUESTION No. 431 Considering the epidemiology of this disease is less frequent which of the following statements? ANSWER a. - may occur in up to 10% of pregnancies b. - Some women may have signs of preeclampsia from week 20. c. - Up to 85% of cases occur in primigravidae. d. - The family history of hypertension.
  • 272. QUESTION NO. 432 Which of conceptional risk factors for preeclampsia is more often seen? ANSWER a. - Recurrent urinary tract infection. b. - Mean arterial pressure equal to or greater than 95 mm Hg. in the second quarter. c. - Excessive weight gain than expected for gestational age. d. - Gestational Diabetes.
  • 273. QUESTION No. 433 Which of the following factors is to develop HELLP poor prognosis for the patient? ANSWER a. - MT 160/110 mmHg. b. - proteinuria greater than 2 g in 24 hrs urine. c. - Creatinine> 1.2 and platelets <100,000. d. - Increased liver enzymes
  • 274. QUESTION No. 434 Which of the following preventive measures have the least evidence to reduce risk? ANSWER a.- Measure blood pressure after the 20th week. b.- Control prenatal nutrition surveillance. c.- Conduct studies of uric acid, creatinine and urea. d.- EGO to search proteunuria (qualitative).
  • 275. QUESTION No. 435 Which of the following preventive measures have the least evidence to reduce risk? ANSWER a.- Avoid stress: a pregnant Although not limited in your daily life if you must have relative rest. b.- Calcium diet: Eat two or three grams of calcium from the first trimester of pregnancy appears to reduce the RISG of developing preeclampsia. c.- Folic acid supplements: This substance is able to reduce the risk of preeclampsia by decreasing the concentration of homocysteine ​in the blood also prevents embryonic malformations such as spina bifida. d.- Diet rich in antioxidants: Vitamin C and E are powerful antioxidants that eliminate free radicals and others that are part of the toxins.
  • 276. CASE REPORT Patient is 19 years old which comes first consultation for secondary amenorrhea for 30 weeks LMP, goes so far as unaware of their pregnancy at home, the patient complained of dizziness, abdominal pain stick, tightness of chest , headache and irritability, physical examination shows hyperemic sclera, mild facial redness, rot's increased and lower limb edema, proteinuria, fundus measured with 28 cm, with decreased fetal activity, the vital signs of the mother were FR 21, FC 96, TA 130/100 3 times.
  • 277. QUESTION No 436 What is the immediate behavior to continue the patient. ANSWER a. - Identify fetal viability. b. - Search for HELLP syndrome. c. - Magnesium sulphate 4 g IV. d. - Prepare the patient for Caesarea.
  • 278. QUESTION No. 437 Which of the following risk factors occurs most often? ANSWER a.- Primiparous or multiparous elderly and Age: <18 or> 35 years b.- Weight: <50 kg or obese (However you should consider that many obese women may have hypothyroidism, polycystic ovarian syndrome, High blood insulin levels and excess estrogen, leading causes of infertility or early abortion). c.- Existence of chronic diseases: diabetes mellitus, hypertension, kidney disease, eye disease, collagen vascular disease (lupus erythematosus). d.- Preeclampsa in a previous pregnancy
  • 279. QUESTION No. 438 Which of the following is less common pathologies as diagnostic difference? ANSWER a.- Epilepsy. b.- TBI. c.- Subarachnoid hemorrhage. d.- Ruptured cerebral aneurysm.
  • 280. QUESTION No. 439 In which of the following conditions can be used magnesium sulphate? ANSWER a.- Torsades de pointes. b.- WPW c.- Priztmetal d.- Brugadra.
  • 281. QUESTION No. 440 Which of the following statements is less common in overdosage of magnesium sulfate. ANSWER a.- Dilated pupils, dry mouth, tachycardia, urinary retention, blurred vision. b.- Behavioral changes, delirium, sedation, hallucinations, conv ulsions. c.- Respiratory depression and coma. d.- Decreased bowel movements and dehydration. Hyperpyrexia by inhibition of sweating.